Torts

Ace your homework & exams now with Quizwiz!

Dr. Dumb, a citizen of State A, performed emergency surgery upon Pam, a citizen of State B, for which he charged $3,000. Pam was injured during the surgery, and she thereafter filed a negligence lawsuit against Dr. Dumb in the appropriate U.S. District Court in State A for $85,000. Dr. Dumb impleads Insurance Co., a corporation incorporated in State B and doing business in States A and B, for payment on his malpractice insurance policy if Pam should win her lawsuit. Which of the following causes of action, if any, may be joined?

A $100,000 counterclaim by Dr. Dumb against Pam for damage done when Pam, angry at the results of the operation, broke Dr. Dumb's office window, if the counterclaim is deemed to be permissive in nature. Discussion of correct answer:If the $1,000 counterclaim by Dumb against Pam is deemed to be permissive in nature (i.e., not arising out of the transaction or occurrence that is the basis of the plaintiff's claim), ancillary jurisdiction would not exist. However, because Dumb's claim does satisfy the "amount in controversy" requirement (it is in excess of $75,000), it does have an independent basis for subject matter jurisdiction, and can be joined in this action.

A woman owned a used bookstore. She was passionate about literature, and spent most of her days behind the counter reading, barely glancing up at customers as they came and went. She had a high employee turnover rate, as her employees usually got frustrated with her lack of management, and, therefore, the general upkeep of the shop was in decline. In fact, there were several cracks in the roof of the shop; over the past few weeks rain had leaked in and formed puddles on the floor, which the bookstore owner noticed but failed to clean. One day a customer came into the store drinking a large soda. He carelessly dropped it on the floor, and the bookstore owner failed to clean it. About four hours later, a second customer came into the bookstore, hoping to find a first-edition of a book. The second customer, trying to break in a new pair of 5-inch heels that were a size too small, could barely walk. As she was teetering up and down the aisles browsing the selection, she slipped on the wet floor and broke her ankle. If the second customer sues the bookstore owner for negligence, why would she prevail?

Because she sued in a pure comparative negligence jurisdiction, and the jury found her 90% responsible and found the bookstore owner only 10% responsible. Discussion of correct answer:The second customer would be able to recover for the portion of her damages for which the bookstore owner was considered responsible (10%). Under a comparative negligence regime, where the plaintiff's negligence has contributed to her own injuries, the total damages caused by the defendant may be apportioned based upon a determination of the relative fault of each party. A majority of states have adopted comparative negligence systems, either by statute or judicial decision. Some states have adopted a "pure" system of comparative negligence. In such states, apportionment of damages tracks apportionment of fault perfectly—if the defendant is 25% responsible and the plaintiff is 75% responsible, the plaintiff recovers from the defendant 25% of the total damages she suffered.

A novice rock climber was acquiring all of the equipment he needed to pursue his new favorite hobby. He saw an advertisement for a sale on Steel Strength heavy duty rope at a sporting goods store. The manufacturer's ad showed a photo of an athletic man standing next to a jagged rock face with a rope tied around his waist in rock-climbing fashion. The ad read: "Steel Strength rope helps me climb where I need to climb and go where I need to go." The rock climber bought a spool of Steel Strength rope at the store. He didn't discuss the rope with the sales clerk because, based on the ad, he was confident that it was what he needed. The climber brought the rope on his next climbing excursion. The climber's large friend, who had also just started climbing, came along, and he borrowed some of the Steel Strength rope. The rope, which really wasn't designed for supporting even an average-weight person, broke under his weight. He fell from the rock face, landing on a park ranger. The friend broke both of his legs, and the ranger broke her arm. The friend and the park ranger sue the manufacturer in product liability for their injuries. Which of the following is correct?

Both the ranger and the friend can prevail against the manufacturer, because the advertisement created an express warranty. Discussion of correct answer:The ad that the rock climber saw, as well as the name of the product itself, amounted to an express warranty that the rope was useful for rock climbing. In addition, anyone who could reasonably be expected to be affected by particular goods is entitled to the benefit of any associated warranties. Both the ranger and the friend could reasonably be expected to be affected by rope that is advertised as useful for rock climbing but is not. They need only show that the defendant made a warranty as to the quality or nature of the product and that the product did not conform to the warranty.

A young man and his friend were cliff jumping from a popular jump site. As the young man approached the edge of the cliff, he said that he saw a person at the bottom. The friend, a more experienced jumper who had introduced the young man to the sport, looked down and said to go jump and that the young man will land nowhere near the person at the bottom. The young man was uncertain, but jumped anyway and lost control of his parachute. A woman was swimming in a quarry near the landing site. The young man released his parachute as he spun over the quarry and landed on top of the woman. The woman lost consciousness, and suffered a broken arm as the young man struggled to drag her ashore. Who bears liability for the accident?

Both the young man and his friend, because they were joint tortfeasors. Discussion of correct answer:The young man is liable for negligence. An actor is required to exercise the care that a reasonable person in his position would find necessary to prevent an unreasonable risk of harm to another. In this case, the young man acknowledged that it was unreasonable to jump seeing a person at the bottom. The friend is jointly liable because he encouraged the young man to commit a negligent act. A person is subject to liability if he or she knows that the other person's act constitutes a breach of duty and gives substantial assistance or encouragement to the other person to breach the duty. In this case, the friend—a more experienced jumper who could reasonably have expected that the young man would follow his advice—encouraged the young man to breach his duty of reasonable care by telling the young man to jump, knowing of the young man's reasonable uncertainty. Therefore, the young man and the friend were joint tortfeasors and both bear liability for the accident.

A winemaker went to a New Year's Eve party at the home of his friend, an electrician. He had already drunk quite a bit of his own wine when he arrived, and soon demanded vodka from the host. After the winemaker started trying to peer down women's blouses and roughly insisted on kissing the electrician's sister under the mistletoe, several more sober guests cornered him and locked him in a bedroom to calm him down at the electrician's direction. When the winemaker stopped beating on the door a half hour later, they let him out. He was still quite drunk. A friend of the winemaker's took him outside, walked him to the bus stop, and told him to go home. The winemaker came back to the electrician's house and tried to re-enter the house trying to seek revenge for the embarrassment of being locked in the bedroom. When he was not allowed to re-enter, he tried to climb up to the balcony. He slipped on some ice on the way up and broke his ankle. What would be the winemaker's best claim for relief against the electrician?

False imprisonment for having him locked in the bedroom. Discussion of correct answer:A person is subject to liability for false imprisonment if he acts intending to confine a person within certain fixed boundaries, and his act directly or indirectly results in such a confinement of the other, and the other is conscious of the confinement or is harmed by it. Here, the electrician directed his guests to confine the winemaker to a bounded area, and the winemaker was aware of the confinement.

A man lent his friend his sterling silver, antique punch bowl for the friend's annual Christmas party. The party was a rousing success. As the man was leaving the party, he told the friend that he needed the bowl back in two days or his wife would have a fit. The friend ended up keeping the man's punch bowl for two weeks. On the date in question a small plane lost power and crashed into the friend's house, causing massive damage and starting a fire. The punch bowl was destroyed in the crash. If the man sues the friend based on negligence, what is the likely outcome?

Finding for the friend, because a plane crashing into his house was an unforeseeable risk. Discussion of correct answer:A superseding cause is an unforeseeable intervening cause that breaks the chain of causation between the initial wrongful act and the ultimate injury, and the friend could not have anticipated the destruction of the punch bowl by the plane crash. Therefore, the destruction of the punch bowl was unforeseeable and was caused by an unforeseeable superseding force. This results in a finding of no proximate cause and no liability.

A pet owner was out on a walk with his dog. While on the walk he was throwing a stick for the dog to retrieve. The dog was not on a leash. It was usually a good dog that was fully trained and obedient. As they were approaching a busy roadway the pet owner threw the stick. As the pet owner was throwing the stick, his neighbor got his attention and started talking to him. Because the pet owner was talking to his neighbor, the pet owner didn't realize that the stick landed in the roadway and therefore didn't call the dog from the road. When the dog ran into the road, a driver of an approaching car slammed on his brakes. The driver's brakes locked up; his car fishtailed across the centerline and hit a woman's car. The woman was severely injured. The driver's car normally would have had anti-lock brakes, which would have prevented the skid and fishtail. However, he recently had brake problems and had his mechanic unhook the anti-lock braking sensor. If the woman sues the pet owner, what is the likely result?

Finding for the plaintiff, because the pet owner should have been controlling his animal. Discussion of correct answer:If the defendant's conduct was a substantial factor in causing the plaintiff's injury, it follows that he will not be absolved from liability merely because other causes have contributed to the result; such causes, innumerable, are always present. The pet owner should have been paying attention to what his dog was doing as he approached a busy roadway. The pet owner contributed to the incident by throwing the stick the dog was chasing into the street. Therefore, the pet owner is liable for the injuries, because his negligence contributed to the events that led to the woman's injuries. An accident was clearly a foreseeable consequence of the pet owner allowing his dog to run into the street.

During Spring break, a student and three of her friends, all students at a local university, decided to take a trip to a resort. They planned to take turns driving so they could drive straight through to the resort without stopping. The student's first turn to drive came when they were driving through a remote part of a national forest. As soon as the student got behind the wheel, she realized that she needed to find a restroom. However, the car had just passed a sign stating that the next public rest area was 60 miles down the highway. After they drove a few more miles, the student saw a sign indicating that a restaurant was located at the next exit. She decided to exit the highway to see if she could use the restroom at the restaurant. As the student entered the restaurant, the hostess directed her toward the restrooms. The student noticed a sign over the doors to the restrooms that read: "Restrooms are for the convenience of our customers only." Although neither the student nor her friends intended to buy any food from the restaurant, the student used the restroom facilities. Then, just as the student was about to exit the restroom, she slipped on a puddle of water on the tile floor and fell, breaking her arm. The puddle was caused by a leak in the drainpipe under the restroom sink. The owner of the restaurant had been notified about the problem by another restaurant customer, who slipped on the same puddle of water two hours before the student's accident. However, the restaurant owner had not yet done anything to address the problem. Which of the following is the best description of the student's legal status?

Invitee. Discussion of correct answer:The restaurant owner had a duty to inspect the premises for non-obvious, unknown dangers, and to warn the student or make the premises safe. Even though the student had no intention of purchasing any food in the restaurant, and even though the restaurant had a sign that restrooms were for customer use only, the student would still be considered an invitee. According to Prosser, a business owner who allows customers to use a restroom on the business premises owes the same duty of care to one who enters the premises solely to use the restroom as is owed to one who intends to make a purchase. An owner or occupier of land owes such invitees a duty to not only inspect the premises for non-obvious, unknown dangers, but also to warn invitees of the danger or make the premises safe. Thus, this is the correct answer choice.

A man owned an auto repair shop where he regularly rebuilt engines for customers and put them into cars that needed new engines. A high school student came in and told the auto repair shop owner that he needed his car to get to school, that it was broken down, and he didn't know what to do. The auto repair shop owner agreed to tow the car to the shop. Once at the shop, the auto repair shop owner realized that he had a Ford LTD engine he recently rebuilt that would be perfect for the student's car. The auto repair shop owner put the new engine into the car. The student picked up the car. The next morning, the student was on the way to school with his friend when the gas pedal got stuck. He crashed into a telephone pole, injuring both himself and his friend. They both missed the last two weeks of school and failed to graduate. If the accident occurred because the connection between the gas pedal and the engine was not properly lubricated when the new engine was installed, and the friend sues Ford under a strict products liability theory for her injuries, what will the outcome be?

No liability, because the problem area had been part of the rebuilding process. Discussion of correct answer:Strict product liability requires that the defect exist at the time it left the defendant's control. In the case at hand, it is clear that the defect resulted from the rebuilding by the auto repair shop owner, and, therefore, the manufacturer is not strictly liable.

A teenage girl and her boyfriend have been dating for several years. The girl's father objects to the relationship and tells several of his friends and acquaintances that if the boyfriend doesn't leave his daughter alone, the father will "tear the boyfriend limb from limb." The father never threatens the boyfriend to his face. The father is a retired professional wrestler who was forced into retirement for using excessive force in the ring. The boyfriend sues the father for assault. Should the boyfriend prevail in his assault case?

No, because a threat of future violence does not constitute an assault. Discussion of correct answer:An action for assault requires the apprehension of an immediate harmful or offensive contact. Here, there is no evidence that the father made threats in a way that would cause a reasonable person to fear an immediate harmful contact. Words alone, even credible threats of future violence, do not constitute assault and are not actionable.

A girl received a coin collection from her father several years ago for her birthday. After a falling out with her father, she decided to sell the coins and buy designer shoes. She did not know the value of the collection, so she took it to a local appraiser. The girl's friend had taken items to the appraiser for appraisal in the past and told the girl she believed he was fair. The appraiser, an avid coin collector, noticed that several coins in the girl's collection were coins that he had been trying to obtain for years. He knew that the entire collection was worth about $10,000, but he offered her $2,000. The girl was ecstatic that her coin collection was worth so much money, but she was turned off that the appraiser was so pushy about purchasing the coins. Despite his insistence, she took her coin collection to be appraised in the next town over. The second appraiser told her the coin collection was worth about $10,000. The girl sues the first appraiser for misrepresentation. Should she recover?

No, because she did not suffer any pecuniary loss due to the first appraiser's statements. Discussion of correct answer:Luckily for the plaintiff, the first appraiser's manner caused her to seek a second opinion and not take any action in reliance on what he said. To recover for misrepresentation, a plaintiff must establish that: (1) the defendant made a false representation, knowing that it was false or with reckless disregard as to whether it was true or false; (2) the plaintiff materially and justifiably relied on the misrepresentation; and (3) the plaintiff incurred pecuniary damages. Thus, this answer is correct. She cannot recover, because she did not suffer any pecuniary loss.

A math student and an architecture student attend classes at the local university. The architecture student's calculus final exam is scheduled for Monday, while the math student's exam is scheduled for the following Wednesday. After his final on Monday, the architecture student travels home to visit his family and does not return to the university until the following week. The morning of her final, the math student realizes that her calculator is broken. Without a working calculator, she is certain to fail the exam. The math student enters the architecture student's unlocked dorm room and removes his calculator from his desk drawer. After the exam, the math student returns the calculator in perfect condition to the precise location in which she had found it. Can the architecture student recover against the math student for trespass to a chattel?

No, because the architecture student cannot prove that he sustained any damages. Discussion of correct answer:A trespass to a chattel may be committed by intentionally dispossessing another of the chattel, or using intermeddling with a chattel in the possession of another. Unlike other intentional torts, the plaintiff must prove actual damages from the intermeddling, or in the case of dispossession, by the loss of use. Actual damages is one of the elements of the trespass to chattels cause of action. Because the architecture student will not be able to show any damages on these facts, this answer choice is correct. Here, the math student took the architecture student's calculator to use in her final exam, so her actions were indeed intentional. However, the architecture student cannot prove that he suffered damages as a result of the math student's actions. When the math student was in possession of the architecture student's calculator, the architecture student had completed his final exam and was visiting his family. There is nothing in the facts to suggest that he needed or wanted to use the calculator during the time that the architecture student was in possession of it, or that the value of the calculator was diminished. Therefore, the architecture student unlikely to prevail in a claim for trespass to a chattel.

The teams in a paintball league usually play with Type A pellets, soft-shelled pellets that pose little risk of injury to players. However, occasionally, when competition begins to heat up, the read team captain like to "kick things up a notch," meaning that both participating teams switch to hard-shelled, Type B pellets, which pose considerable threat of injury, even when players wear protective gear. The blue team member has played several paintball games against the red team captain and her team. The blue team member always wears protective gear. However, the blue team member has never played in a particularly heated match against the red team. One weekend, the blue team member was playing paintball against the red team. The score was close, and the red team captain asked the blue team member if she wanted to "kick things up a notch." The blue team member responded affirmatively, not realizing the significance of the red team captain's question. When the game resumed, the red team captain and her team had donned protective gear and switched to Type B pellets. When the first pellet hit the blue team member's armor, her right arm was severely bruised. The blue team member filed a claim against the red team captain for battery.

No, because the blue team member had given her express consent. Discussion of correct answer:Where a defendant has otherwise committed a tort, consent by the plaintiff may work as a defense. Consent is defined as the plaintiff's willingness in fact that the defendant's action take place. Consent may be granted orally or it may be implied from the plaintiff's conduct. As a regular participant in the paintball league, the blue team member clearly consented to the regular risks of the sport. Furthermore, the blue team member expressly consented to the red team captain's suggestion that they "kick things up a notch." In cases of intentional torts, express consent remains valid even when granted by mistake, unless the defendant knows of or takes advantage of that mistake. Here, the blue team member was wearing her protective gear as usual, and her injury occurred shortly after the teams resumed the game using the hard-shelled, Type B pellets; thus, there were no external indicators to the red team captain that the blue team member did not expect the shift from soft-shelled to hard-shelled pellets. As such, although the blue team member granted her consent without realizing the implications of the red team captain's question, it does not appear that the red team captain knew of or took advantage of the blue team member's mistake. The blue team member would not prevail in a battery claim, because the actions of the red team captain would be excused through a successful defense.

A city employee and an engineer rent a house in the city. The city employee has worked for the city for 15 years without taking a sick day. The engineer, always the responsible one, tends to be very solicitous of the city employee and worries when the city employee goes to work while sick. One day, the city employee awoke with a terrible flu. The engineer begged the city employee to stay home, but the city employee did not listen. Instead, the city employee decided to take some cold medicine and left for work. While driving to work, the city employee became drowsy from the medication and crashed into a woman's car. Due to the crash, the woman driver sustained injuries to her head and neck. Is the woman driver likely to prevail in a negligence action against the engineer?

No, because the engineer had no duty to control the city employee's actions. Discussion of correct answer:In this case, it was the city employee, not the engineer, who crashed into the woman's car. Thus, the engineer will be liable to the woman driver only if he had a duty to control the city employee's actions. Generally, (1) there is no duty to control a person's conduct to prevent him from causing physical harm to a third party absent a special relationship that imposes a duty upon the actor to control the other person's conduct; or (2) a special relationship exists between the actor and the other person, giving the other person a right to protection. The city employee and the engineer are roommates, which does not create a special relationship that imposes a duty on the engineer. Because the engineer had no duty to control the city employee's actions, the woman driver cannot recover in a negligence action against the engineer. This is the correct answer choice.

A woman owned a pet tiger cub, despite the "No Pet" rule in her apartment building. Most of the time, she kept the cub locked in her guestroom, which had no windows, so neighbors would not see the cub from the street. However, she occasionally let the cub loose to play around her apartment, so that he wouldn't get depressed by being confined to one room. The cub generally acted like a house cat, and when he was allowed out of the guestroom, he spent most of his time sleeping on a corner of the couch. The cub was very friendly, but unaware of his strength and sometimes bit too aggressively when playing with chew toys. One evening, the woman invited her friend over to watch a movie. The friend was aware that the woman had a pet tiger cub, but he did not know that she let it out of the guestroom occasionally. When the friend arrived, the woman forgot to mention that the cub was sleeping on the couch. While the woman was in the kitchen making popcorn, the cub woke up and wanted to play with the friend. The cub grabbed its favorite chew toy, carried it over to the friend and dropped it at his feet. Annoyed, the friend started batting the cub across the face with the toy. The cub snapped at the friend, biting his finger. The friend jumped backward and fell over the edge of the couch, breaking his arm. If the friend sues the woman for his injury, should he prevail?

No, because the friend knowingly and unreasonably subjected himself to a risk of harm. Discussion of correct answer:An owner of a wild animal, even a wild animal kept as a pet, is strictly liable for injuries caused by a dangerous propensity common to the species of animal in question. However, the victim must do nothing voluntarily to cause the animal to injure him. The fact that the friend knew that the woman owned a tiger cub does not provide the woman with a defense. The friend did not know that the cub was not in its room. However, assumption of risk is a defense to strict liability when the plaintiff appreciates the danger and voluntarily subjects himself to the danger. Once the friend discovered that the cub was out of its room, he batted its face with the toy, causing the cub to bite him. Therefore, the friend unreasonably and voluntarily took on a known risk. Since the friend thereby assumed the risk of injury, he cannot recover from the woman for his injury.

A man bought a barbecue grill from a local store. The store only sold barbecue grills and did not manufacture them. One week later, the man noticed that the hose that ran from the propane tank had worn thin and the grill did not heat up as quickly as it should. The man was annoyed at this development, but he had a backyard full of hungry guests and continued to cook. One hour after he first noticed the worn hose, the man began to smell propane. He continued to cook, because he had only one hamburger left on the grill. Just as the man was finishing, the hose burst, and propane began rushing out of the tank. The propane ignited, seriously burning him. Will the man prevail in a strict liability action against the store?

No, because the man knew that the hose was worn. Discussion of correct answer:Strict products liability is invoked against manufacturers, suppliers, and distributors when a defective product for which defendant is responsible injures an appropriate plaintiff. Factors that govern whether strict liability applies include the defective nature of the product, the nature of injuries suffered, whether the parties to the suit are proper, and whether any defenses are available. However, if the plaintiff discovers or is warned of the dangerous defect and voluntarily and unreasonably continues to use the product, such continued use is usually regarded as assumption of the risk and bars the plaintiff's recovery in strict liability. Thus, if the man voluntarily continued to use the grill with knowledge of the danger, he will be barred from recovery in strict liability.

A man liked his privacy, and had special dark-tinted windows installed in his car, despite a city ordinance prohibiting tinted windows darker than a certain shade--two shades lighter than the windows that the man had selected. One day, he was driving down a busy street, sunroof open, music blaring, and windows down. When he stopped for a red light, he took the opportunity to light a cigarette. The man rested his elbow on the ledge of the open window. Meanwhile, a woman was just leaving a local coffee shop. Realizing she was late for an important appointment, she sprinted out of the shop into the street in order to cross before the light changed. In her haste, the woman bumped into the man's car and also collided with his lit cigarette, which did not burn her severely, but startled her enough to step back--directly into the cars coming from the opposite direction. As a result, the woman was hit by an oncoming car and sustained severe injury. She filed suit against the man to recover her medical expenses, asserting negligence per se based on the man's violation of the tinted window ordinance. Is the woman likely to prevail?

No, because the man's violation of the ordinance was not a cause of the woman's harm. Discussion of correct answer:Pursuant to the negligence per se rule, a defendant's violation of a criminal statute has the effect of establishing the standard of care only when all of the following three conditions are present: (1) the injury caused by the defendant's conduct was the type that the statute was intended to prevent; (2) the plaintiff is a member of the class intended to be protected by the statute; and (3) the defendant's violation of the statute was not excused. By adopting the statute or ordinance as the standard of care, the violation of that statute will be considered sufficient to prove breach of duty (though the plaintiff will still need to prove causation and damages). In this case, the woman will be unable to establish negligence per se. The statute in this instance was a criminal one, and the woman, as a pedestrian, may be within the class intended to be protected by the statute. However, she will be unable to establish the first requirement of negligence per se. The harm which occurred was in one sense similar to the type of harm the statute was designed to prevent, but the facts indicate that the incident occurred while the man's vehicle was motionless at a traffic light, and not as a result of anyone's impaired vision due to the man's tinted windows. As such, the man's violation of the ordinance would not constitute negligence per se and would not suffice to establish the man's duty and breach of duty, making this the best response.

The last surviving member of a wealthy family lived on the largest parcel of residential real estate in town. He had had a falling out with his longtime best friend. Just prior to the wealthy man's death, the friend had returned to town, and they reportedly made amends. After the wealthy man's death, the friend bought the property. A reporter for the local newspaper was waiting in the office of his real estate lawyer when he saw the friend coming out of a conference room, shaking hands with the lawyer. Curious, the reporter went to the county clerk's office where he knew all deeds were recorded and learned that the friend had acquired the estate at a price substantially below the original asking price. The next day, the front page headline read, "Friend 'Steals' Estate for $250K." When the friend saw the headline, which was accompanied by his picture and photos of the late wealthy man and the main house on the estate, he became incensed, inasmuch as he had hoped to keep his return to town secret. Will the friend succeed in an invasion of privacy action against the newspaper?

No, because the newspaper printed public facts. Discussion of correct answer:Based on the facts presented, the friend's invasion of privacy action must lie in the tort of public disclosure of private facts. This form of invasion of privacy consists of the defendant's unreasonable public disclosure of private facts about the plaintiff. Given that all the information published was public information contained in recorded deeds, the friend cannot prevail in his action.

A foreign tourist had never seen an American football game. He visited the United States and happened upon a neighborhood park where local residents were playing a game of football. The tourist watched the game for one quarter. At the start of the second quarter, one of the players was tackled and injured, and the tourist volunteered to take the injured player's place. During the time remaining in the game, the tourist successfully tackled several players from the opposing team. With less than 30 seconds to go in the final quarter, the tourist caught the football passed to him by the quarterback and was promptly tackled by three players from the opposing team. He suffered several bruises and a sprained wrist. Will the tourist prevail in a battery claim against the three players?

No, because the tourist participated as a player for several quarters. Discussion of correct answer:Where the defendants have otherwise committed an intentional tort, they are not liable in tort if the plaintiff consented to the act and the defendants did not exceed the scope of that consent. Consent may be granted expressly, in words, or it may be implied from the plaintiff's conduct. Here, the tourist volunteered to take the place of a player who had been injured when tackled. The tourist then actively participated in the game for almost three full quarters, and he himself tackled several opposing players during the course of the game. A reasonable person would interpret the tourist's conduct as consent to be tackled. Therefore, this answer choice is correct.

An attorney was dissatisfied with her position at a local law firm. As a result, she applied for a position with another local law firm. The hiring attorney at the law firm to which the attorney applied, noticed on the attorney's resume that she was currently employed at a law firm at which one of the hiring attorney's law school friends was a partner. The hiring attorney then proceeded to call the partner for a reference. When asked by the hiring attorney about the attorney's competence, the partner replied that his impression was that her work was less than satisfactory. Although the partner had no personal knowledge of the attorney's work, he believed that to be a fair assessment of her competence, because one of the other partners had in fact said that about her. In fact, though, the partner's negative opinion of the attorney's work was based on one episode which later turned out to be the fault of another attorney. Because of the partner's adverse comment regarding her qualifications, the attorney was not employed by the hiring attorney's firm. Will the attorney be successful in asserting a defamation claim against the partner?

No, if the partner's statement was based on a reasonable belief regarding the attorney's competence. Discussion of correct answer:An employer has a conditional privilege against defamation when he or she comments on an employee's performance at the request of a prospective employer. This privilege exists when a defendant communicates on a matter of interest to the recipient. This answer choice correctly states the law—the partner is not liable if his statement was based on a reasonable belief. The privilege, since it is conditional, can be defeated by a showing that the employer's opinion was based on actual malice, that he exceeded the scope of the privilege, or that he does not believe the communication to be true. If any of these applies, the defendant is liable for defamation. On the other hand, if his opinion was based on reasonable (albeit mistaken) grounds, his opinion is not actionable defamation. Since it does not appear that the defendant's opinion was based on malice, that he exceeded the scope of the privilege, or that he did not believe the statement to be true, he is not liable.

For several weeks, a boy and his neighbor, who were both 11 years old, had been working together to build a go-cart. When the go-cart was finally finished, the boys couldn't wait to try it out. They designed the go-cart so that both of them could ride on it at the same time. The boys also figured out how to make a crude steering mechanism by attaching a length of rope to the front axle. They wanted to try the go-cart on a street with a slope. The boy and his neighbor headed over to a cul-de-sac, the only street in their neighborhood with a substantial incline. When they reached the top of the incline, the boys jumped on the go-cart and took off, very quickly picking up speed. At the bottom of the incline, a busy street crossed the cul-de-sac. As the boys neared the bottom of the slope, they noticed that a car driven by a man was about to enter the intersection between the cul-de-sac and the busy street. The boy, who was riding behind his neighbor, saw the man's car coming. Because he believed that the neighbor would not be able to stop or turn the go-cart in time to avoid a collision, he yelled, "Jump off!" When the neighbor didn't move, the boy pushed the neighbor off the go-cart. The neighbor suffered a broken wrist and severe scrapes on his face and right arm. The go-cart, with the boy still on it, shot into the intersection. The boy was struck by the man's car and suffered multiple fractures and internal injuries. If the neighbor's parents, on behalf of the neighbor, sue the boy's parents to recover damages for the neighbor's injuries, will they win?

No, unless the boy acted unreasonably when he pushed the neighbor off the go-cart. Discussion of correct answer:An individual, when engaged in any activity, has a legal duty to act as a reasonably prudent person under the circumstances. Note that a defendant has a duty to act as a reasonably prudent person would in the particular circumstances under which the defendant acted. The existence of an emergency does not relieve the individual from the responsibility of acting reasonably. Minors are expected to act in a manner that is similar to how a reasonable child of the same age, education, intelligence, and experience would have acted. Given the facts of the question, this is the best answer, and the neighbor's parents will not recover damages from the boy's parents unless the boy acted unreasonably when he pushed the neighbor off the go-cart.

A boy and his friend were two 10-year-old classmates. They worked everyday after school on rebuilding an old bicycle. When the bike was finally finished, the boys could not wait to take it for a test ride around the block. Both wanted the honor of being the first to ride it, so they decided that the boy would sit on the seat and the friend would ride on the handle bars. As the boys turned up on a city street, a driver was driving his truck toward them. He was trying to eat a messy sandwich while driving, and he was not paying attention to the road. The two boys saw him coming, but they both knew there was no way to avoid being hit. The friend, frozen in panic, would not respond when the boy urged him to jump from the handle bars. The boy then pushed the friend to the ground at the last minute. The driver hit the boy, causing severe injuries. The truck did not hit the friend, but he suffered a broken arm when the boy pushed him from the bicycle. If the friend's parents, on behalf of their son, sue the driver, will they prevail?

No, unless the driver was driving negligently when the collision occurred. Discussion of correct answer:The most logical theory under which the friend's parents would sue the driver is negligence. Thus, this is the best answer, because the friend's parents will not prevail in a negligence action unless the driver was driving negligently when the collision occurred.

An accomplished seamstress decided to go into business making dresses for weddings. A bride-to-be saw one of her advertisements and called to see if she would be interested in making a wedding dress for her. They decided to meet at a local fabric store to discuss patterns and fabrics. The bride-to-be mentioned that her five-year-old niece was going to be the flower girl at her wedding. At the end of the meeting, the seamstress suggested that the bride-to-be come to her house the following Saturday to take her measurements. She also asked her to bring her niece along to play with her six-year-old granddaughter while they took measurements. The bride-to-be agreed to bring her niece with her and the next Saturday, they both went to the seamstress's house. The seamstress took the bride-to-be into the sewing room, and the niece and granddaughter went outside to play in the sandbox. Which of the following best describes the legal status of the bride-to-be and the niece after they entered the seamstress's house?

The bride-to-be was an invitee, and the niece was a licensee. Discussion of correct answer:An invitee is a person who enters onto the property of an owner of land in response to an invitation by the owner for a purpose that relates to the owner's business interests. The invitation may be express or implied. In contrast, a licensee is a person who enters onto the property of the owner (or occupier) with permission of the owner for the person's own purposes or benefit. Permission may be express or implied. Social guests are considered to be licensees. Under the facts of the question, the bride-to-be, as the seamstress's customer, was on the premises in connection with the seamstress's business interests. On the other hand, the niece, although invited to be on the premises by the seamstress, would be best categorized as a social guest. Her presence was not connected to the seamstress's business interests. Thus, this is the best answer.

A fraternity president lived in a fraternity house with 25 other fraternity brothers. The fraternity had been known for years for its weekly wild keg parties. One of the better known features of the party was that at the end of the party all of the empty beer kegs were stacked up one on top of the other as evidence of the fraternity brothers' drinking prowess. This was a fraternal tradition. Every new fraternity brother was required to help the president stack the empty kegs. This week a new brother accepted the honor of stacking the kegs. The next morning the kegs came crashing down on a mailman who delivered mail to the fraternity house, breaking several bones in his foot. The mailman sued the new brother. Which if the following is correct?

The brother should be held liable, even though he was acting in a manner consistent with a fraternity custom that had gone on every week for years. Discussion of correct answer:Although acting according to a custom can be a defense to a charge of negligence, it is not conclusive. The trier of fact may find either that the customary manner of behavior was not reasonable under the circumstances or that a reasonable person would not have engaged in customary behavior. A trier of fact will probably find both that the custom of stacking kegs on top of one another was not reasonable and that a reasonable person would not engage in such dangerous behavior.

A businessman owned a lumber mill business that used by-products from other mills to make pressed-board products. The process created a sludge that required disposal. The businessman had been dumping it into a large pond on the property. However, this long-term dumping had caused the pond's water to become contaminated. This pond has now attracted swarms of mosquitoes and flies to the pond and the surrounding vicinity. As a result, a professor, who is an adjacent landowner, has found it nearly impossible to go outside due to the high numbers of flying insects now swarming about outside. The professor is now unable to let his young children play outside because of the presence of numerous mosquitoes due to a concern for West Nile virus. The professor then filed a nuisance action against the businessman for his use of the pond to dump the sludge. How is the court likely to rule?

The businessman's act is a nuisance, because it unreasonably interferes with the professor's use of his property. Discussion of correct answer:A private nuisance is a thing or activity which substantially and unreasonably interferes with a plaintiff's use and enjoyment of his land. The interference with a plaintiff's use and enjoyment must be substantial; this means that it must be offensive, inconvenient, or annoying to an average person in the community. Here, given that the businessman's use of the pond unreasonably interfered with the professor's use and enjoyment of his property, the professor is entitled to sue for private nuisance.

A cautious driver drove down the street at a speed just below the posted limit. A speeder was traveling in the opposite direction, approaching the cautious driver at a high rate of speed. Suddenly, the speeder saw that a red car was stopped at an intersection, waiting to make a left turn. Realizing that he would not be able to stop in time to avoid colliding with the red car, and seeing that several children were bicycling to his right, the speeder veered across the center line and into the cautious driver's lane. The only action the cautious driver could take to avoid colliding with the speeder was to steer his car onto a homeowner's lawn. How, if at all, is the cautious driver liable to the homeowner?

The cautious driver is not liable to the homeowner for trespass, but is liable for any damage caused to the homeowner's property. Discussion of correct answer:Trespass to land is an intentional act that causes a physical invasion of the plaintiff's land. In cases involving a "private" necessity, such as an imminent emergency, a private party is privileged to trespass upon the property of another. Here, as the cautious driver was trying to avoid a collision with the speeder's car, he was privileged to enter the homeowner's land by driving onto the lawn. However, the cautious driver would be liable for any damage he caused to the homeowner's lawn. As such, this is the best answer.

A convenience store sold specialty frozen drinks in a variety of flavors, including sugar-free. One weekend, a new employee mistakenly filled the sugar-free frozen drink machine with the regular sugary frozen drink mix. A diabetic surgeon came in to purchase his standing favorite, a large raspberry sugar-free frozen drink. Due to the employee's mistake, the surgeon consumed far more sugar than his system could tolerate. Within an hour of drinking the sugary drink, he was feeling woozy and lapsed into a diabetic coma shortly thereafter. He was admitted to the hospital and successfully treated, but he continued to experience blurriness of vision, which doctors were hopeful, but not certain, would improve in time. A world-renowned trumpet player who was suffering from carpal tunnel syndrome had been scheduled for surgery with the surgeon, whose specialty was operations to address carpal tunnel syndrome. However, given the surgeon's medical problems, the surgeon was unavailable to perform the surgery as scheduled, and the trumpet player found another surgeon to perform the surgery. However, the surgery failed to improve the trumpet player's condition, and following the surgery, he had lost all mobility in his wrist and was unable to play the trumpet. A production company had hired the trumpet player to play trumpet on an upcoming movie soundtrack, pending his surgery. Based on the trumpet player's famous name, the production company had lined up numerous large investors in the film. When the trumpet player came out of the surgery unable to perform on the soundtrack as planned, the company's investors backed out of the deal, causing the company major financial losses. The company filed a claim against the convenience store seeking recovery of its severe financial losses. The jurisdiction applies a comparative negligence standard. Which of the following is the strongest argument in defense of the convenience store?

The convenience store is not liable for the company's damages, because the company's damages were too far removed from the store's negligent act. Discussion of correct answer:A defendant's duty of care is owed only to foreseeable plaintiffs‚ i.e., those within the risk of harm created by defendant's unreasonable conduct. In the same vein, the doctrine of proximate cause acts as a limitation on liability based on the extent to which the risk created by the defendant's unreasonable conduct will allow recovery for the harm suffered by the plaintiff. The Restatement 2d approach allows a defendant's liability to be cut off where the court finds highly extraordinary the relationship between the defendant's conduct and the resulting harm. Alternatively, many jurisdictions extend liability only to the foreseeable consequences of defendant's negligence. In this instance, there exist a number of factual uncertainties, including whether the trumpet player's surgery would necessarily have succeeded if performed by the surgeon and the uncertainty of financial investors in general. While it is possible that a court might still find that the damages suffered by the company were sufficiently linked to the negligence of the convenience store as to make the store liable, the store's strongest defense lies in its assertion that the plaintiff and the damages suffered were so unforeseeable as to make it inappropriate for the convenience store to bear liability for the company's injury. As such, this is the best response.

A homeowner hired a decorator and a painter to work on his house. The painter was at the top of a 12-foot ladder painting an upper portion of the house, when the decorator needed the ladder as well. Not paying attention to what she was doing, the decorator grabbed the ladder out from under the painter's feet without looking up to see whether the painter was using it. The painter screamed for help. The homeowner was watching a football game on television and had been drinking beer all day. Hearing the cry for help, the homeowner looked out of the second-story window to find the painter hanging by his fingers high above the ground. The homeowner told the painter to hold on. The homeowner's plan was to take the painter by both hands and lower him to the ground. This was a ridiculous idea and would never work, but the homeowner overestimated his own strength due to his drunken condition. The decorator almost had the ladder back under the painter's feet when the homeowner grabbed the painter's hands from the window sill and dropped the painter to the ground below. The painter broke both legs and sued the decorator. Which of the following is correct?

The decorator should be held liable for negligence. Discussion of correct answer:The decorator should be held liable, even though the homeowner's attempted rescue was negligent. The homeowner's rescue attempt was a foreseeable intervening force resulting from the decorator's negligent conduct. An intervening force is one that actively operates to produce harm after the defendant has already committed his negligent act. A defendant is liable if the intervening force is normal or foreseeable. Intervening forces that are normal or foreseeable and that will support a finding of proximate cause include a negligently executed rescue. The homeowner's failed rescue attempt was, therefore, a normal and foreseeable intervening force, supporting a finding of liability on the decorator's part.

A real estate developer specializes in the renovation of historical buildings. His current project is the conversion of a Victorian school campus into condominiums. The oldest building had been modernized 30 years ago when a four-story steel fire escape had been installed. The developer, a trim man of substantial vigor for his age, was doing a lot of the exterior painting himself to save money. He greatly improved the appearance of the old building by painting the fire escape an attractive forest green. When renovations were finished, members of the local fire department were called to inspect the premises and issue a smoke detector certificate. The fire inspector was a large, heavy-set man. For public relations purposes, he conducted his inspections in full firefighting uniform—big black boots, steel hat, and heavy lined raincoat. The uniform added 35 pounds to his weight of 250 pounds. The developer showed the inspector around the building. As part of his inspection, the inspector climbed the fire escape to evaluate this means of egress from the apartments on the upper floors. When he reached the second floor landing, the painted stair rail gave way, causing the inspector to fall and seriously hurt himself. Examination revealed that underneath the new green paint, the fire escape was badly rusted and could not support the inspector in his uniform. If the inspector sues the developer for his injuries, who will prevail?

The developer, because he was not aware of the dangerous condition of the fire escape. Discussion of correct answer:This is the best answer, because it correctly states the developer's required standard of care toward the inspector. Although the developer invited the inspector onto the premises for the developer's business purposes, courts traditionally classify the inspector as a licensee because he is a firefighter. (This also applies to police officers.) The developer's duty to a licensee is to fix or warn of known dangers. He is not required to make his land safe for licensees and is not required to inspect to discover dangers, even if a reasonable inspection would have revealed them. If the developer was aware that the fire escape would be dangerous to someone the inspector's size, its condition would be a known defect about which the developer must issue a warning. Nothing in the facts shows that the developer knew of the unsafe condition.

When an advice columnist for a local paper received a letter from "Terribly Teary-eyed," he knew that the author was his ex-girlfriend. His response to her letter was published in the evening edition and referred to "Terribly Teary-eyed" as a completely immature person who must not know how to have a normal relationship because of her experience as a prostitute. When the ex-girlfriend read the newspaper and the response to her letter, she was mortified. Shortly thereafter, the ex-girlfriend found out that the columnist was her ex-boyfriend. She immediately sued him and the newspaper. The columnist moved to dismiss. If the court grants the columnist's motion to dismiss which of the following, if true, is the most likely reason why?

The ex-girlfriend cannot show that she is "Terribly Teary-eyed," and that others knew her by that identity. Discussion of correct answer:When suing for defamation, a plaintiff must prove that some reasonable third person who received the defamatory message understood it to refer to the plaintiff. If the ex-girlfriend could not show that others recognized her as "Terribly Teary-eyed," she will not be able to prevail on a claim for defamation. The ex-girlfriend must also show that the publication damaged her in the eyes of the community and that the publication was not merely opinion, but was couched as a statement of fact.

A noted paleontologist was working on an important dig in a remote area in the western United States. In the middle of their planned stay on location, her group was fortunate to find a nearly intact skeleton of an infant velociraptor. Anxious to determine if there were adult specimens nearby, the group redoubled their efforts to expand the dig. The paleontologist hired a local excavator to do the initial soil removal at the new location. The excavator had no experience at dinosaur digs and was not accustomed to working with many people near his backhoe. One afternoon, the excavator moved a pile of rocks to a ledge above the original dig. A large boulder later rolled off and hit a volunteer with the paleontologist's group who joined through his hometown science museum. The volunteer was severely injured and will never walk normally again. The volunteer sued the paleontologist and the excavator in a jurisdiction which has adopted a contribution rule based on degrees of fault, but has retained joint and several liability. The jury found that the volunteer's damages were $100,000, the volunteer was not negligent, the paleontologist was 20 percent at fault, and the excavator was 80 percent at fault in causing the volunteer's injuries. After the excavator paid the volunteer $100,000, he sued the paleontologist for reimbursement. What is the likely result?

The excavator will recover 20 percent of the total damages, because he and the paleontologist are jointly liable for the volunteer's injury. Discussion of correct answer:This is a negligence question, where the defendant must pay damages for a breach of a duty to the plaintiff. According to the facts, this jurisdiction has adopted a rule of contribution based on degrees of fault. Because the defendants are jointly and severally liable and the paleontologist is 20 percent at fault, she will be required to reimburse the excavator for 20 percent of the judgment.

The plaintiff was visiting a friend when the friend's pet dog walked into the room. The plaintiff was frightened of the dog and tried to leave the room, but the dog was blocking the only doorway out of the room. The plaintiff then tried to distract the dog so he can leave the room but only succeeded in startling the dog, who bit him. Which of the following arguments would best support the plaintiff's suit against his friend under a strict liability theory?

The friend knew that the dog bit someone last week. Discussion of correct answer:This is the best argument that the plaintiff can make on these facts. Domestic animals have "known dangerous propensities" only if a reasonable owner would realize that the animal presented a danger of death or injury. The classic example of a domestic animal with known dangerous propensities is a dog who has previously bitten someone (this is known as the "one bite" rule because, after one bite, the owner is presumed to know that the dog is dangerous). If the dog in this case did indeed bite someone last week, the defendant will be presumed to know of the dog's dangerous propensities and therefore be strictly liable to the plaintiff, so this answer choice is correct.

A chauffeur lived next door to a well-known gang member with a long criminal record, including theft and violent crimes. The gang member had robbed the chauffeur's house twice, but the authorities were not able to convict him. Recently, the chauffeur was robbed again. He actually had no idea who the thief was, but suspected the gang member. The chauffeur called the police and told them that he saw the gang member running from the chauffeur's house carrying the stolen goods. The police arrested and prosecuted the gang member. However, the gang member went free when the judge was forced to declare a mistrial because a juror was doing independent research about the case. The gang member sues the chauffeur for malicious prosecution. Which of the following is the most accurate?

The gang member should not prevail, because his case was dismissed on a technicality. Discussion of correct answer:To prevail on malicious prosecution, the case against the plaintiff must have terminated in a manner indicating the plaintiff's innocence. Malicious prosecution is the institution of criminal proceedings by a defendant, done for an improper purpose, and without probable cause, which terminate favorably to the plaintiff, and which cause the plaintiff damages. The criminal prosecution must have terminated in a fashion indicating that the plaintiff was innocent of the charges. Termination on the merits (acquittal after trial, court dismissal for lack of sufficient evidence) is sufficient in this regard; termination based on procedural or technical defects, prosecutorial discretion, or similar grounds is not. A mistrial resulting from juror misconduct is a technical defect that does not indicate the gang member's innocence. Therefore, the gang member should not prevail in his action.

An avid gardener specialized in wild vines, ivies, and shrubs. The crowning glory of her garden was a towering trellis, covered with green vines and ivies, which abutted her neighbor's backyard. The neighbor had a two-year-old son who loved to play in the backyard. The son was extremely allergic to poison ivy. Unbeknownst to the gardener, poison ivy had taken root in her garden. Given all the other varieties of ivy, the gardener failed to notice the poison ivy. One afternoon while playing in the backyard, the son brushed up against the gardener's trellis and unknowingly came in contact with the poison ivy. Within five hours, his skin had broken out in blisters and itchy bumps. The neighbor (his mother) treated him as best she could with calamine lotion and told him not to scratch. She intended to take him to the doctor in the morning, if necessary, where the doctor could give him an injection to stop the swelling. During the night, however, the son's itching became unbearable, and he could not stop himself from scratching and scratching until his arms and legs were a bloody mess. The next morning when she came in to check on the son, the neighbor was horrified to find her young son unconscious in his bed; he'd grown weak and passed out from loss of blood. She took him to the emergency room, only to learn that the son was a hemophiliac, a medical condition which the son's pediatrician had not yet had reason to discover. The son was admitted into the children's intensive care unit, where he remained for several weeks. Although he finally recovered, the neighbor incurred thousands of dollars in medical expenses and lost wages. The neighbor files suit against the gardener to recover her financial damages. Which of the following arguments offers the gardener her best defense?

The gardener should not be held liable, because the sprouting of poison ivy in her garden is not probative of a breach of due care. Discussion of correct answer:In an action for negligence, the plaintiff has the burden of proving that the defendant failed to conform to a standard of conduct appropriate for a person of the defendant's education and experience. It may be inferred that harm suffered by plaintiff is caused by negligence of the defendant only when (1) the event is of a kind that ordinarily does not occur in the absence of negligence; (2) other responsible causes, including the conduct of the plaintiff and third persons, are sufficiently eliminated by the evidence; and (3) the indicated negligence is within the scope of the defendant's duty to the plaintiff. Here, these elements are not satisfied. For one, it is questionable whether the existence of poison ivy in the gardener's garden is indicative of a breach of reasonable care on her part, inasmuch as poison ivy and other unwelcome plants may crop up anywhere in any lawn or garden despite a gardener's impeccable care. Thus, although the son's and, therefore, the neighbor's, injury was caused by poison ivy, it is not clear that the growth of the poison ivy was the result of a breach of due care on the gardener's part. As such, the gardener's best defense is that the harm suffered by the son was not the result of a breach of due care on the gardener's part.

A homeowner's house was blown up on orders of the fire chief of a town in order to prevent the spread of a fire to the entire town. The homeowner was prevented from removing any personal goods from the house before it was destroyed. Assuming the fire chief's conduct was reasonable, will the homeowner prevail if she sues the town form conversion?

The homeowner will not prevail because the fire chief was privileged to act as she did to prevent greater harm. Discussion of correct answer:This situation involves the destruction of property that has become harmful to the community. A person's right to the use and enjoyment of his property is subject to the state's lawful exercise of its police power. The exercise of the police power involves the destruction or limitation on the use of property that, under the circumstances, is a danger to the public safety, health, or welfare. Where the state exercises its police power, the landowner is not entitled to compensation. In this case, even a private individual would be privileged to destroy the plaintiff's house, and the owner of the property would not be entitled to compensation, since it is given that the defendant's conduct was reasonable in this case.

A homeowner owned a backyard above-ground swimming pool. One day, she noticed that the pool filtration unit did not seem to be functioning properly. She determined that she needed a new filtration unit, so she went to the local pool supply store to purchase one. The homeowner looked for the model number for the filtration unit designed for her particular brand of pool, but noticed the store was out of that model. The homeowner saw another filtration unit that looked similar to the one she currently had. Although it was not manufactured specifically for her brand of pool, the homeowner decided it was close enough, so she purchased the filtration unit and took it home. The homeowner installed the filtration unit, carefully following the manufacturer's directions. When the installation was complete she activated the filtration unit. Because of a defect in the electrical system, the filtration unit shorted out, causing a spark that burned the homeowner's hands. In an action by the homeowner against the manufacturer, who will prevail?

The homeowner, because she was injured by the defective electrical system. Discussion of correct answer:When an injured plaintiff sues a product manufacturer in strict liability, all she needs to show in order to prevail is that the product was defective and that the product was not substantially changed in the course of its distribution from manufacturer to consumer. Because the electrical system was defective, and there is nothing in the facts to suggest that the pool filtration unit was substantially changed before the homeowner purchased it, the homeowner can sue the manufacturer under a strict liability theory. The fact that it may not have been the best unit for her pool is not relevant.

A villager walked through his village one winter night. The temperature was well below freezing. As he was walking, he saw a man in nice clothes lying in an alley, which was unheard of in his upscale village. Because it was so out of the ordinary, he went over to the man to investigate. When the villager tapped the man, he saw that the man was wearing an expensive suit and was clearly intoxicated. Disgusted, he left the man lying in the alley. About an hour later, a teenager led the police on a chase on his dirt-bike. He led the police down the alleyway. The teen was able to avoid the man still lying in the alley, but the police hit him. When they checked the man, he was dead. When the villager heard of this he told the police his story so the officers involved wouldn't feel guilty. Two days later, the autopsy revealed that the man was dead from cold exposure prior to the police hitting him. Under which of the following situations, if true, is the villager most likely to be found liable to the man's estate?

The man was a patron at the villager's bar an hour earlier. Discussion of correct answer:Where a defendant has a special relationship with the plaintiff, he has a duty to aid the plaintiff. The relationship between a business and its patron is usually considered to be one such relationship. If, as this answer choice states, the man was a patron at the villager's bar, then the villager had a position of control over the plaintiff, so he is bound to aid him. The villager could have stopped serving the man but did not; therefore, he would have a duty to protect the man from harm.

A nature lover brought all kinds of animals and plants to her three-acre farm and let them roam wherever they wanted. The animals lived on the property for years without incident. Her longtime neighbor wanted to sell his adjacent farm but had no luck because of the noise the animals made. Frustrated, he filed a nuisance action against the nature lover, seeking damages for the decrease in value to his property as a result of her animals. What will be the result of the neighbor's lawsuit?

The neighbor will not prevail, because the presence of animals in farming areas is not unreasonable. Discussion of correct answer:A private nuisance is substantial and unreasonable interference with a plaintiff's use and enjoyment of her land. To recover in a nuisance action, the interference with a plaintiff's use and enjoyment must be "substantial," that is, it must be offensive, inconvenient, or annoying to an average person in the community. A plaintiff cannot, by devoting her land to an unusually sensitive use, make a nuisance out of conduct that would otherwise be relatively harmless. Here, the neighbor lived adjacent to the nature lover's property for a long time and had no complaint about her animals until he was unable to sell his land. Moreover, inasmuch as they lived in a farming community, it is unlikely that the nature lover's conduct would be offensive, inconvenient, or annoying to an average person in the community. Whether the nature lover kept the animals well away from the property lines will not matter to the court as much as whether the animals produced some type of significant interference with the neighbor's use or enjoyment of his land. Because it is not unreasonable to raise animals on a farm, it is unlikely that the neighbor can prevail in a nuisance action.

A hardware store owner sold power tools. A salesman for a certain brand of power tools came to the store and convinced the owner to discontinue his relationship with his current tool company and to exclusively carry his company's tools. The salesman told the owner that his tools were more powerful and therefore better than the other brand. He told the owner that a television handyman had recently signed a contract to serve as their celebrity spokesman and gave the owner a large sign to hang in his store advertising his power tools as the only tools good enough for the celebrity handyman. Relying upon the salesman's representations about his tools and the handyman, the owner bought the tools, hung the sign in the store, and removed the other brand of tools from his sales floor. Two weeks later, the owner discovered that the entire line of his new power tools had been recalled, because their motors were not powerful enough to qualify as power tools within the government's definition of the term. In fact, the motors were less than 25 percent as powerful as the other brand of tools the owner had removed from his shelves, a fact of which the salesman had been aware long before he met the owner. The owner also received a summons and complaint from the celebrity handyman's attorney who had filed suit for invasion of privacy, because the handyman actually was a spokesman for another brand of power tools. The owner filed suit against the salesman for misrepresentation. Who will prevail?

The owner, because he was justified in his reliance on the salesman's representations. Discussion of correct answer:To establish a prima facie case of intentional misrepresentation, a plaintiff must prove that the defendant's representation was false and made with scienter (knowledge of its falsity or reckless disregard of the truth), and that the plaintiff materially and justifiably relied on it and suffered damages. Reliance on a misrepresentation of fact is generally regarded as justified, except where the representation is patently false. The plaintiff has no duty to investigate, even where it would not be burdensome to do so. If the plaintiff actually investigates, the plaintiff may not rely on representations inconsistent with what he actually or might reasonably have discovered. Here, because all of the elements of misrepresentation are present, the owner will succeed because his reliance on the salesman's statements was reasonable.

A tabloid reported that a billionaire real estate developer and casino mogul who was considering running for president had recently been treated for chronic alcoholism at the inpatient clinic of a large hospital. In reality, the developer had actually undergone a series of cosmetic procedures to enhance his appearance in order to improve his chances of winning his party's nomination and, ultimately, the presidency. Two days after the story broke, a senator who was the highest ranking elected official in the developer's upstart party, while engaged in a filibuster, began reading the lead stories from newspapers all across the nation, including the tabloid article. The senator is an outspoken supporter of the developer's main rival for the party's nomination. What is the likely result if the developer sues the senator for defamation?

The senator will prevail, because he was privileged to make the statements. Discussion of correct answer:Where an absolute privilege applies, a defendant may not be held liable for an otherwise defamatory message as a matter of law. A legislator (federal, state, or local) is not liable for a defamatory message uttered while on the floor of the legislature or during hearings or committee proceedings. The nature or content of the defamatory message or its relationship to any matter before the legislature is immaterial to availability of this privilege. Here, because a senator made the statements on the floor of the Senate, the absolute privilege applies, regardless of whether the statements were defamatory.

A university student's family had immigrated to the United States only a few years ago. The student had experienced a great deal of war and violence in his native land. Upon the recommendation of a fellow student, the student signed up for an eight-week meditation course at a local meditation center. The course involved a 40-minute sitting followed by a "dharma" talk. The first week that the student attended the course, he found the meditation period uncomfortable and difficult. He approached his instructor to ask whether it was normal for "difficult feelings" to arise during meditation. His instructor assured him that, indeed, this was normal, particularly for beginning meditation students, but that the sittings would most likely become more comfortable with patience and time. The following week, during the sitting, the student found himself so overwhelmed with formerly repressed memories of past trauma that by the end of the session he left the class wordless. Upon returning home, he continued to spiral into despair. Several hours later, he slit his wrists and bled to death. The student's family filed suit against the meditation center, claiming that the instructor had failed to adequately warn the student of the risks of meditation. They sought damages for the student's wrongful death, and their own pain and suffering. Which of the following statements regarding the student's family's claim is most accurate?

The student's family will not prevail, unless it is shown that the meditation instructor had reason to know that the student's background placed him in extreme risk of suicidal depression as a result of meditation. Discussion of correct answer:The establishment of a right to recover damages resulting from a party's negligent act is often complex and fact-intensive. However, the basic elements of any negligence claim remain constant. To recover in an action for negligence, a plaintiff must show: 1) that the defendant owed the plaintiff a particular duty of care; 2) that the defendant breached that duty of care; 3) that damages resulted; and 4) that the defendant's negligent act was the legal or proximate cause of those damages, or as otherwise stated, that the plaintiff's damages were a foreseeable consequence of the defendant's negligent act. Here, as the student's teacher, the meditation instructor certainly owed the student a duty of reasonable care—that is, to conduct himself according to a standard appropriate for a "reasonable" meditation instructor. From the facts presented, there is no indication that the meditation instructor acted less than reasonably. Rather, from the facts, it appears that the student's death was so unforeseeable and far removed from the incident in question (when the instructor assured the student that difficult feelings during meditation were normal), as to make it quite unlikely that the student's death can be considered a foreseeable consequence of the instructor's actions or inaction. However, if the family were to show that the instructor had reason to know that the student had an especially traumatic background and faced a risk of resurfacing traumatic memories during meditation, and a corresponding risk of suicidal thoughts, it is possible that the family could succeed in establishing that the instructor's actions (or, more likely, inaction) fell short of the duty of care he owed the student. Thus, this is the best response.

A handyman used his snowmobile on the weekends for recreation. One day, he and his friend were using the snowmobile and crashed into a large snowbank. Both men were thrown from the snowmobile, which ran over both of them. The handyman was killed and the friend was so seriously injured that he had to have his leg amputated. Statutes require that all personal snowmobiles have a termination switch, which is supposed to shut off the motor on impact. An investigation revealed that the switch had fallen off the snowmobile when the plastic piece inside the switch snapped in half, which is why the snowmobile failed to stop upon impact with the snowbank. The snowmobile manufacturer bought switch assemblies for the termination switch from a supplier. The supplier had a stringent testing program for the assemblies, and their testing records from that shipment showed no defects detected during the testing. If the friend files a negligence suit against the supplier, under what facts would he be most likely to recover?

The termination switch was defective, and a reasonable inspection by the supplier would have uncovered the defect. Discussion of correct answer:The potential danger involved in the use of snowmobiles makes it necessary for the manufacturer to inspect its components and the final product. For defective products, a plaintiff suing in negligence must show the existence of the defect and that the defect would have been discoverable upon reasonable inspection. If a reasonable inspection would have found the defect, and the supplier did not reasonably inspect, this would lead to a finding for the plaintiff under a negligence theory of product liability.

A city was experiencing a severe water shortage. To mitigate the problem, the city enacted an ordinance limiting total water use and prohibiting the use of water by private citizens for any purpose other than that of personal hygiene, cooking and personal consumption. Use of water for landscaping, gardening, washing cars and the like was strictly prohibited and was punishable by fines of up to $5,000. A homeowner had a thriving vegetable garden which required daily watering. Since she limited her personal water use, she felt justified in limited use of sprinklers to water her garden. The sprinklers were timed to water the garden between 6:00 and 6:20 p.m. every night, before the homeowner got home from work. As her garden was in her backyard, the homeowner was relatively confident that her water use would go unnoticed by town officials, and it did. However, the sprinklers did not escape the attention of a boy and his friend, the seven- and eight-year-old boys who lived next-door to the homeowner. Noting that the homeowner's sprinklers went off like clockwork every night at 6:00 p.m., just when temperatures in the desert town were peaking, the boy and his friend made a habit of putting on their bathing suits and sneaking into the homeowner's backyard to run through the sprinklers, unbeknown to their mother and to the homeowner. One evening, in his enthusiasm, the boy, tripped over the sprinkler spigot and scratched his ankle. The wound went unnoticed by his mother until several days later, when she noticed that her son was limping. She took him to the doctor, only to learn that he had developed a staph infection and required immediate hospitalization. The boy was hospitalized for over a month before he recovered sufficiently to be released, but his infected leg remained weak and his medical bills were extensive. The boy's mother filed suit against the homeowner to recover her medical expenses and lost wages, citing the homeowner's violation of the ordinance. The court accepted the statute as written. Which of the following statements is most accurate?

The unexcused violation of a legislative enactment or administrative regulation which is adopted by the court as defining the standard of conduct of a reasonable man, is negligence itself, or as commonly phrased, negligence per se. The violation of a criminal statute constitutes negligence per se where the plaintiff can show that: 1) he was a member of the class of persons the statute was designed to protect; and 2) he suffered the type of harm the statute was designed to prevent. In such instances, statutory violation will be considered sufficient to prove duty and breach of duty, but a plaintiff is still required to show causation and damages. Here, the doctrine of negligence per se does not apply. For one, the statute involved was a civil ordinance, not a criminal statute. And secondly, the type of harm suffered by the boy (an injury occurring when tripping over a sprinkler spigot) was not type of harm that the ordinance was designed to prevent. As such, the boy's mother will not prevail on grounds of negligence per se.

A wife and a husband were extremely pleased with their new home. The wife was ecstatic because the house had a huge yard with a tennis court. The wife was an avid tennis player who had always wanted to teach her husband to play. The husband, who has never excelled at sports, was skeptical but willing to give it a try. However, their first lesson was not a complete success; the husband hit half of the tennis balls into the net and the other half into the metal fence surrounding the court. Finally, the husband hit a ball over the fence, through a stand of tall pine trees and into the yard of their next-door neighbor. The neighbor was in her garden cutting roses for a flower arrangement at the time tennis ball entered her yard. The ball landed in a nearby birdbath, splashing the neighbor with water. Since the neighbor did not see the ball coming, the water startled her, causing her to drop her clipping shears onto her foot, which bruised her big toe. If the neighbor sues the wife and the husband for trespass, which is the most accurate statement of their liability?

The wife and the husband are not liable for trespass, because they did not intend to case the tennis ball to enter the neighbor's yard. Discussion of correct answer:The tort of trespass to land is defined as an intentional physical invasion of the plaintiff's possessory interest in land caused by the defendant. The defendant does not have to enter onto the land; a defendant may be liable for trespass if he causes a physical object to go onto the land. In addition, no actual injury to the land is required. However, intent is required. Under the facts presented, the wife and the husband are not liable for trespass, because they did not intend to cause the tennis ball to enter the neighbor's yard. Thus, this is the best answer.

A hair spray manufacturer recently shipped a batch of hair spray that had far too much alcohol added to it, which causes the spray can to catch fire far too easily. A woman bought a can of the hair spray. The can plainly read: "DO NOT USE AROUND OPEN FLAME." Nevertheless, the woman proceeded to spray her hair while smoking a cigarette. The spray ignited and burned the woman's face. The woman was rushed to the hospital. Which of the following is correct?

The woman should recover, because the product was defective. Discussion of correct answer:A product manufactured in a form other than the manufacturer intended contains a manufacturing flaw. Here, the hair spray was defective in that it contained too much alcohol. All jurisdictions impose strict liability on the manufacturer and all other sellers of the product in the chain of distribution for personal injury or property damage to plaintiff caused by a manufacturing flaw. The warning here did not prevent the product from being unreasonably dangerous. Furthermore, a foreseeable misuse does not bar recovery under products liability, even if a warning label was placed on the product. (It may reduce recovery in some jurisdictions.) Thus, the fact that the woman did not heed the written warning does not bar her recovery, because the manufacturer could expect that some people would smoke while using the product. As such, the woman should recover in strict products liability.

A woman used a new facial cream that was the biggest new craze in the city and could be purchased only at the salon where it was made. Due to a manufacturing error, the cream contained a large amount of bleach, which burned the woman's face. The woman sued the salon at which she purchased the cream. The salon defended on the grounds that it had exercised due care. What is the likely outcome of the woman's lawsuit?

The woman will prevail, because the salon manufactured and sold the cream. Discussion of correct answer:The seller of a defective product in a condition that is unreasonably dangerous to the user or consumer or to his property is strictly liable for harm caused by the product if the seller is in the business of selling such a product, and if it is expected to and does reach the consumer without a substantial change in condition. This rule applies even if the seller used all possible care in the preparation and sale of his product, and the consumer has not bought the product from or entered into any contractual relation with the seller. Here, the salon created and sold the cream, so it is strictly liable for the manufacturing defect.

A developer builds a group of 200 recreational homes, in four basic models, in a resort area. The homes feature high-beamed ceilings, indoor spas, ski storage rooms, lake views, and fireplace inserts. A woman bought one of the builder's units, which the county inspector had already found in compliance with the building code. She moved into her new home in the fall. Temperatures were getting cool at night, so she stoked up a fire she thought would last for several hours and warm the house, then went to bed. However, a defect in the chimney allowed exhaust smoke to seep into the upstairs bedroom where she was sleeping, causing her moderate injury from smoke inhalation. The woman sues the builder. Who will prevail?

The woman, because a defect in the residence caused her injury. Discussion of correct answer:In most jurisdictions, the owner-developer of tract housing is subject to strict liability if an injury results from an unreasonably dangerous defect in a house he constructed. The builder of a single home, either custom-built or to be sold on the market, is not subject to strict liability. The builder built 200 units in this development, from only four different models, and will be held to be a tract seller.

At a conference concerning the criminal justice system in a populous state, a prominent criminal defense lawyer who has practiced in most of the district courts of the state, made a statement that the district judges of the state can all be bribed and that money can buy a defendant an acquittal. A district court judge who has never presided over a matter in which the lawyer participated, has sued the lawyer for slander. Which of the following would be the lawyer's best defense?

There are 250 district court judges sitting in the courts of the state. Discussion of correct answer:A statement directed to a group or class of persons can be "of or concerning" a particular person only if the group or class is small enough for the statement to be reasonably understood as referring to the plaintiff, who is a member of the group, or the circumstances give rise to the conclusion that there is a reference to a particular member of the group. If the group defamed is sufficiently large, then there is no cause of action for slander in any single member of the group. Since 250 judges would constitute a very large group, this is the lawyer's best defense.

A man and a woman were investment bankers who had known of each other for some time. They went for a business lunch at a trendy little place packed with other financial professionals. The woman said, "I've heard so many good things about you. What would it take to get you to come work at my office?" The man yelled, "I would never work with you! You're an unethical lowlife." Several other financial professionals who were eating lunch overheard his reply. Will the female investment banker prevail in a defamation suit against the male investment banker?

Yes, because it was reasonably foreseeable that someone would overhear the statement. Discussion of correct answer:To prevail in a defamation action, a plaintiff must establish that a defendant published defamatory material concerning that plaintiff, which caused damage. The message must be communicated to a third person. While the defamatory message must be understood by the person who receives it, the recipient need not believe the defamatory message or have a rational reaction. If it is reasonably foreseeable that a third party might overhear a defamatory message, and one does so, there is sufficient publication to bring suit. Given that the male investment banker yelled out his defamatory remarks at a crowded lunch spot, it was reasonably foreseeable that his statement might be overheard. Therefore, there was publication, and the female investment banker is likely to prevail.

A technical support employee of a computer company spent his morning traveling the city providing in-home tech support to multiple customers. The last scheduled home repair visit took much longer than normal, and the employee telephoned the office to ask his boss whether he would mind if the employee went home for lunch rather than return to the office before making his final series of tech support calls in the late afternoon. The employee's boss agreed, and the employee drove home to eat lunch. While at home, the employee drank several beers with his meal. Realizing he was too intoxicated to drive, the employee had his precocious 10-year-old daughter drive him to his next tech support call. The daughter drove him safely across town but, upon entering the client's driveway, she crashed into the client's car. Will the computer company be liable for the accident?

Yes, because the employee's negligence will be imputed to his employer. Discussion of correct answer:An employer is liable for his or her employees' tortious acts that are committed within the scope of employment and that cause injuries or property damage to a third person. If an employee negligently entrusts performance of his work to a third person, the employer may be liable by imputed negligence. In this case, it was the daughter who was driving the car, and there is nothing to indicate that she was an employee of the computer company. However, given that the employee breached his duty of due care by having his young daughter drive him to his next service call, the employee's negligence may be imputed to his employer, and the computer company will be liable for the resulting damages. As such, this is the best answer.

A husband and wife were walking down a country road when they were frightened by a steer running loose on the road. To avoid the steer, they climbed over a fence to get onto the adjacent property, which was owned by a farmer. After climbing over the fence, the husband and wife damaged some of the farmer's crops that were growing near the fence, on which was posted a large "No Trespassing" sign. The farmer was walking with his large watchdog when he saw the husband and wife. The farmer approached them, hoping to frighten them with the fierce-looking dog, but the leash broke and the dog bit the wife. If the wife files a battery claim against the farmer, will she prevail?

Yes, because the farmer intended for the dog to frighten the wife. Discussion of correct answer:To be liable for battery, a defendant must act with the intent to cause an immediate harmful or offensive contact, or know that such a contact is substantially certain to occur. Here, the farmer acted to cause the wife immediate apprehension of an offensive contact, which is the intent required for assault, not battery. However, under the doctrine of transferred intent, if a defendant acts with the intent to commit one tort against a victim and ends up committing a different tort against that victim, the intent from the first tort will "transfer" to the second tort, which will be sufficient for liability. The doctrine of transferred intent applies to assault, battery, false imprisonment, and trespass to land or chattels. Thus, in this question, although the farmer intended only to commit an assault, his intent "transfers" to the tort of battery, which is the tort that he ultimately committed when his dog bit the wife. Thus, the intent requirement is satisfied, and the farmer is liable for battery.

An amateur violinist dreamed of one day playing professionally. She attended a performance of a prestigious symphony orchestra. Arriving early, she was surprised to discover that several of the musicians' instruments had been placed at the foot of the stage and were unattended. Wanting only to know what it was like to play such a fabulous instrument on a ground stage, she picked up the violin and carried it to the middle of the stage, planning to play just a few notes and return it to where she'd found the violin. It so happened that a friend of the violinist that owned the violin had also arrived early and saw the amateur violinist pick up his friend's precious violin. The friend rushed onto the stage and snatched the violin from the amateur violinist's hands, causing her to lose her balance and fall. The amateur violinist broke her wrist in the fall and was unable to play the violin to school children. She filed suit against the friend for her medical expenses and other damages. Will the amateur violinist prevail? A No, because the friend was privileged to use reasonable force to recapture the violin. B No, if the friend's use of force was reasonable. C Yes, because the amateur violinist's taking was not wrongful. D Yes, because the friend was not the owner of the violin.

Yes, because the friend was not the owner of the violin. Discussion of correct answer:The use of reasonable force against another for the purpose of reception of a chattel is privileged if the other: (1) has tortiously taken the chattel from the actor's possession without claim of right; (2) has otherwise tortiously taken the chattel from the actor's possession and is about to remove it from the premises; or (3) has received custody of the chattel from the actor and refuses to surrender it or is about to remove it from the actor's premises. Under the majority view, the actor may recapture a chattel only where the original taking was wrongful. Here, the amateur violinist took the violin from the violinist's possession; thus, under the requisite circumstances, the violinist would be privileged to recapture the chattel (i.e., the violin). However, the friend was not the violin's owner and is not privileged to recapture the chattel. Additionally before using force, the defendant must demand that the chattel be returned. The friend did not issue such a demand before using force. As such, this is the best response.

A homeowner was very upset to learn that the old empty school house across the street was going to be renovated into a care house for unwed teenage mothers. The homeowner, a strict believer in abstinence until marriage, was repulsed by the thought of living across the street from unwed mothers, and went around the neighborhood with a petition to halt the renovations. To his dismay, not only were most of his neighbors not bothered by the prospect of the care house, but many of them thought it was a good idea. Nonetheless, the homeowner protested to the city council, arguing that the building should be once again used as a school. Despite his protests, the city council allowed the care house to proceed with renovations as planned. One warm afternoon, as the homeowner was sitting on his porch reading the newspaper, he noticed three young and noticeably pregnant women moving into the building. They were all 6-8 months pregnant and had come to the care house to have their babies in a supportive environment. The homeowner, outraged by his new neighbors, started standing on the sidewalk outside their house with a camera, daring the young women to show themselves in public and trying to photograph them. The young women sued the homeowner for invasion of privacy. Will the young women prevail in their action?

Yes, because the homeowner intruded on the young women's seclusion. Discussion of correct answer:One type of invasion of privacy is intrusion into seclusion. That appears to have happened here, as the homeowner's intrusive conduct would be highly objectionable to a reasonable person. Therefore, this is the best answer.

A motivational speaker had several business trips planned for the summer months. She asked her friend to watch her house while she was away. The motivational speaker gave her friend the master key to her home, which could be used to open all the doors to the house and the garage. The motivational speaker left June 1 for her trip, planning to return home on August 1. On the morning of June 15, the friend used the master key to enter the home. While inside, she turned on the television and noticed a commercial for a theme park that had introduced a new ride the previous weekend. The friend was disappointed, because her car was in need of repair and not in a condition to make the long drive to the theme park. The friend decided to borrow the motivational speaker's car to drive to the theme park. She used the master key to enter the garage, where she found the car with the keys still in the ignition. The friend then drove the car to the theme park to enjoy the new ride. While enjoying the rides at the theme park, the friend received a call on her cell phone from the motivational speaker, who had returned home earlier than scheduled for an important business appointment that day and demanded to know the whereabouts of her car. The friend promised to return the vehicle promptly. The theme park was a five-hour drive from the motivational speaker's home. Because she lacked access to her automobile, the motivational speaker missed her business appointment and lost a loyal client and his large account. Is the motivational speaker likely to recover in a claim against the friend for trespass to a chattel?

Yes, because the motivational speaker suffered actual damages from the intentional dispossession. Discussion of correct answer:A trespass to a chattel may be committed by intentionally dispossessing another of the chattel, or by using or intermeddling with a chattel in the possession of another. The plaintiff must prove actual damages as an element of the trespass to chattels cause of action. Here, the friend borrowed the motivational speaker's car, an intentional act that dispossessed the motivational speaker of her car when she needed it. Given that the motivational speaker lost a business client and his large account as a result of the friend's dispossession of the vehicle, it appears that the motivational speaker will be able to prove actual damages. Thus, the elements of the tort are present, and the motivational speaker is likely to prevail in her claim.

A farmer leased a large country estate for a period of five years. One bright sunny day, a year after the farmer had moved onto the estate, the farmer's neighbor was flying a kite in the neighbor's backyard. The wind picked up and sent the neighbor's kite flying onto the farmer's estate, where it became entangled in the limbs of one of the estate's large trees. The farmer discovered the kite and called the neighbor about retrieving his kite from her tree. However, unable to afford to pay someone to climb the immense tree to retrieve the kite, the neighbor decided to simply abandon the kite and refused to remove it. Will the farmer prevail in an action against the neighbor for trespass to land?

Yes, because the neighbor failed to remove the kite. Discussion of correct answer:One is subject to liability to another for trespass to land, even without harm to another's legally protected interest, if he intentionally enters land in the possession of the other or causes a thing or a third person to do so; remains on the land; or fails to remove from the land a thing that he is under a duty to remove. Here, given that the wind caused the kite to fly onto the estate, the neighbor did not intentionally cause the kite to enter the estate. However, since the neighbor owned the kite, he had a duty to remove it from the property upon the farmer's request. Because he failed to remove from the land a thing that he was under a duty to remove, he will be liable for trespass to land.

A baker herniated a disk in his back when a chef negligently knocked the baker down a flight of stairs. The baker's accident occurred over a year ago, but the baker has experienced severe pain in his mid-back since the accident. The baker brought suit against the chef. The baker recently added a bartender as a defendant to the suit because when the baker was walking the bartender's dog, the baker's foot caught in a hidden hole in the bartender's front yard, and when the baker fell, he broke his ankle. Had the baker not suffered the prior back injury, he would have easily regained his balance and would not have been injured. He now has even more severe back pain and needs at least two surgeries to repair the ankle. Assume all the claims are procedurally sound. Should the chef bear any liability for the broken ankle suffered by the baker?

Yes, because the original back injury is a proximate cause of the ankle injury. Discussion of correct answer:When a tortfeasor negligently causes injury to another, the tortfeasor is responsible for all damages proximately flowing from the original injury. Here, we are told that the ankle injury would not have occurred but for the prior back injury. This means that original injury is an actual cause of the second injury. However, actual cause is not sufficient to justify a recovery against the chef. If the baker recovers from the chef, it must be because the original back injury is a proximate or legal cause of the ankle injury. Thus, this is the best answer.

A homeowner granted an easement to the local water district to install a water main through her property. The water main was installed using modern technology and functioned well for many years. However, the water district failed to maintain the system. The district never checked for leaks, flushed the water main system, or cleaned the pipes. Many years later, now a very old woman, the homeowner noticed a small amount of water in her basement. Because the neighborhood was usually very dry, the homeowner realized the water leak must have come from the nearby water main. However, having grown somewhat paranoid as a result of her mild dementia, the homeowner feared that if she contacted the water district, the authorities would deem her too old to be living independently and would force her into a nursing home. For this reason, the homeowner never contacted the water district and simply hoped against hope that the water leak problem would disappear. However, the leak continued to undermine the foundation of her house and eventually created a large hole in the basement wall through which water rushed freely. At the sound of the rushing water, the homeowner hurried down to her basement to rescue some of her more valuable possessions, but it was too late--her valuable artwork and prize oriental rugs had been ruined. With the help of a legal advocate at the local senior center, the homeowner files a suit against the water district for the damages she incurred. Will the homeowner prevail?

Yes, because the owner of the easement is responsible for reasonable maintenance and repairs of any improvements made to the easement. Discussion of correct answer:The owner of an easement has a duty to maintain the easement. In this case, the water district failed to maintain the water main, thereby causing the flooding in the old woman's basement that damaged her homeowner's property. The water district will, therefore, be held liable for the homeowner's damages. This is the correct answer option.

An organic-dairy farmer continually argues with his next-door neighbor, a conventional vegetable farmer who found the organic farmer self-righteous in his concern for the environment. One day, as the vegetable farmer was treating his garden with gaseous pesticide, he impulsively leaned over the fence separating the two properties and blasted one of the organic farmer's cows with pesticide, injuring the cow and contaminating an area of grass. The organic farmer filed an action against the vegetable farmer to trespass to land. Will the organic farmer prevail?

Yes, because the pesticide caused damage. Discussion of correct answer:One is liable for trespass to land, regardless of whether he causes harm to the interests of another, if he intentionally enters land in the possession of another or causes a thing or person to do so. Historically, trespass required the entry of some tangible physical object, but modern courts have extended trespass liability to intrusions caused by microscopic particles or invisible gases, where harm is caused. Here, the vegetable farmer caused an invisible gas to enter the organic farmer's property. Because it was a gas and not a tangible physical object that the vegetable farmer used to invade the organic farmer's property, the organic farmer must show that damages resulted to recover in a trespass to land action. In this instance, the cow and some pasture sustained damage, so the organic farmer will be eligible to recover.

A photographer learned that the mayor of his major city had made an appointment at the veterinary office where the photographer's niece worked. The photographer's niece told him that the mayor was planning to have his ailing 16-year-old dog euthanized. The photographer, an ardent supporter of the mayor, recognized this as a priceless opportunity to boost the mayor's re-election campaign. The mayor was known to be smart and capable, but had been often been criticized as cold and unfeeling. The photographer was convinced that a personal piece about the politician and his dying dog would show the public that this was a candidate with a heart. The next day, the photographer lingered in the veterinary waiting room until the veterinarian came to lead the politician and his dog into a back room. Several minutes later, the photographer saw the vet exit the room, shutting the door behind her. The photographer then opened the door and began snapping pictures of the tearstained mayor stroking the dead body of his beloved pet. At the sound of the camera, the mayor looked up and yelled, "What the hell are you doing? Get out of here!" The photographer tried to explain that he was trying to help the mayor's chances of winning re-election, but the mayor was furious. Shocked at the mayor's reaction, the photographer fled the room. The incident left him so rattled that he gave up on writing the article. The mayor filed suit against the photographer for intrusion upon seclusion, an invasion of privacy tort. Is the mayor likely to prevail?

Yes, because the photographer's act would be highly offensive to a reasonable person. Discussion of correct answer:The tort of intrusion into seclusion requires the intentional intrusion upon the solitude of another and that the intrusion be highly offensive to a reasonable person. In this case, both elements are present. The photographer clearly intruded upon the mayor's solitude, and his intrusion would be highly offensive to a reasonable person. The situation was also one in which the mayor had a reasonable expectation of privacy. As such, the mayor is likely to prevail.

A mother took her children to school and walked the youngest to her first-grade classroom. She stayed and observed the class for an hour, as parents did almost every day. The roadway and sidewalk by the school entrance were very icy, and there had been an automobile accident shortly before school opened. Concerned for the safety of the children and employees, the principal decided to lock the front door so that no one would walk out to the road by the entrance. There were several exits in the rear of the building, which opened on the fenced-in schoolyard. These exits were in the coatrooms attached to each classroom and were not visible from the lobby or the classrooms themselves. All the children, teachers, and staff knew where these exits were. When the mother was ready to leave the school, she tried to exit by the front door, the only entry she had ever used. When she found it locked, she tried unsuccessfully to locate someone to open it. Meanwhile, the principal and staff were all in the basement searching for rock salt they had not used since last year to de-ice the entryway steps. After the mother had waited 15 minutes by the door, the school secretary came upstairs, saw her, and let her out. On the advice of her husband, a former law student, the mother sues the school for false imprisonment. Will the mother prevail in her suit?

Yes, because the principal should have known there might be parents in the building. Discussion of correct answer:If the defendant knows with substantial certainty that his actions will cause confinement of a person, the requisite intent is achieved. The defendant need not know the identity of the particular plaintiff. In this case, parents were observing classes almost every day, so the principal should have known that parents were likely to have been there.

A man bought a ready-cooked chicken and some bread at the supermarket. He returned home, where he was joined by a friend for dinner. The friend was injured when she bit into the chicken and encountered a piece of broken glass. The friend sues the store. Will she prevail?

Yes, because the store is strictly liable. Discussion of correct answer:One who sells any product in a defective condition unreasonably dangerous to the user or consumer or to his property is strictly liable for physical harm thereby caused to the ultimate user or consumer, or to his property, if (1) the seller is engaged in the business of selling such a product, and (2) it is expected to and does reach the user or consumer without substantial change in the condition in which it is sold. This rule applies even where the seller has exercised all possible care in the preparation and sale of his product, and the user or consumer has not bought the product from or entered into any contractual relation with the seller. Here, the store was in the business of preparing and selling chicken to its customers and, by extension, their meal companions. As such, the store is strictly liable for the harm caused to the man's friend.

A pharmacist owned a drugstore in a small town. A saleswoman for Air d'Elegance Perfumes stopped in with a kit full of samples. She told the pharmacist that a supermodel who appeared on magazines around the world and was known as a role model for young women was going to be the new face of Air d'Elegance. The pharmacist liked the perfumes and was a fan of the supermodel, so he allowed the saleswoman to put up a display of the perfumes featuring a beautiful poster of the supermodel. A month later, the pharmacist received a summons and complaint from the supermodel's lawyer. The supermodel had filed a claim for invasion of privacy against Air d'Elegance and multiple retailers. She claimed that she had never signed a contract with Air d'Elegance and that they were using her likeness without permission to promote their product. Will the supermodel prevail in her suit against Air d'Elegance and the pharmacist?

Yes, because the supermodel did not give permission to use her likeness. Discussion of correct answer:One who uses without authorization the name or likeness of another for commercial advantage (to promote a product or service) is subject to liability for the invasion of privacy tort of appropriation of identity or likeness. There is no intent element to this tort, so it is irrelevant that the pharmacist believed that the supermodel's photograph in the display was authorized. Here, all of the elements of appropriation of identity are present, so the supermodel will prevail in her claim.

An actor hired a plastic surgeon to perform a facelift. During the surgery, the surgeon became angry at the scrub nurse and slapped her hand away as she tried to hand him a scalpel. The scalpel flew out of the nurse's hand, cutting the actor on the shoulder. The surgeon was able to suture the resulting injury in such a way that the actor was left with no visible scar or permanent injury. The actor filed a battery claim against the surgeon. Will the actor prevail?

Yes, because the surgeon's intent to make offensive contact with the scrub nurse was transferred to the actor. Discussion of correct answer:The tort of battery occurs when a defendant intentionally causes harmful or offensive contact with the plaintiff, or something closely connected to the plaintiff. That intent can be transferred where the defendant intended harmful or offensive contact against one person but the resulting harm is caused to the plaintiff. Here, the surgeon intentionally slapped the hand of the nurse, resulting in harmful contact with the actor. The surgeon's intent to make offensive contact with the scrub nurse is transferred to the actor, and the surgeon's action constitutes battery not only on the nurse but also on the actor.

A woman owned a pet iguana. Most of the time, she kept the iguana in a glass terrarium. However, she occasionally let the iguana loose to roam around her apartment. Generally, the iguana just picked a spot under some piece of furniture and stayed there. For an iguana, it was relatively well behaved and seldom bit. One evening, the woman invited her friend over for dinner. The friend was aware that the woman had a pet iguana, but he did not know that she let it out of its terrarium from time to time. When the friend arrived, the woman forgot to mention that there was an iguana wandering around her apartment. While the woman was in the kitchen, the iguana decided to check out the visitor and ambled across the floor toward the friend. In his haste to avoid being bitten, the friend tripped over the coffee table and fell, breaking his wrist. If the friend sues the woman for his injury, should he prevail?

Yes, because, as owner of the iguana, the woman is strictly liable for the friend's injury. Discussion of correct answer:An owner of a wild animal, even a wild animal kept as a pet, is strictly liable for injuries caused by a dangerous propensity common to the species of animal in question. Of course, the victim must do nothing voluntarily to cause the animal to injure him. Under the facts presented above, this is the correct answer, because the friend's injury resulted from his attempt to avoid being bitten. Biting is a dangerous propensity common to iguanas.

A businessman owned an old strip mall. He had done well over the years, but recently his vacancy rate was up, so he decided that it was time to renovate. He contracted with a design company to design and upgrade the facades of the shops to coincide with the Victorian revitalization that was occurring in the town. The company was a licensed general contractor in the town and had an excellent reputation. A week after the grand reopening, a woman was walking out of one of the shops when a section of concrete trim broke loose and fell, hitting the woman and causing a fractured skull. If the woman recovers from the businessman, does the businessman have any recourse against the design company?

Yes, if the finding against the businessman was based on vicarious liability. Discussion of correct answer:If two people are liable in tort to a third person for the same harm and one of them discharges the liability of both, he is entitled to indemnity from the other if the other would be unjustly enriched at his expense by the discharge of the liability. The businessman could be entitled to indemnification from the company if the finding against the businessman was based on vicarious liability, but the businessman would have to prove that the company's work was defective and the businessman failed to discover the defect.

A hiker enjoyed hiking and mountain climbing in remote parts of the world. One weekend, the hiker and her friend decided to climb one of the largest and most formidable mountains in the region. The hiker and the friend were familiar with some of the mountains in the area, but they had never before hiked the largest mountain. As the hiker and her friend were climbing a particularly challenging section of the largest mountain, the hiker lost her footing and slipped. She fell 25 feet before finally landing on a rock shelf below, but she broke both of her legs. The hiker's friend immediately went for help, and alerted the mountain rescue team that the hiker was stranded on the mountain. The mountain rescue team used a helicopter to locate the hiker, and decided that the only way to remove her from the mountain was to airlift her. After the helicopter arrived at the site of the hiker's accident, an experienced rescuer from the mountain rescue team was lowered down to the hiker with a specially designed stretcher. The rescuer carefully moved the hiker into the stretcher. A harness manufactured by a local harness company was attached to the stretcher so the hiker could be lifted into the helicopter. As the hiker was being lifted into the helicopter, the latch on the harness unexpectedly released. The hiker and the stretcher fell back to the mountain, and the hiker suffered a broken arm and foot. If the hiker files suit against the mountain rescue team and the rescuer, she will most likely

ecover, if the harness latch was not properly fastened to the stretcher. Discussion of correct answer:Under the facts presented, the hiker's cause of action against the rescuer and the mountain rescue team would be in negligence. (Inasmuch as neither the rescuer nor the mountain rescue team were in the business of selling harnesses, the hiker would not have a strict product liability claim against them.) Because the rescuer worked for the mountain rescue team, the hiker would be able to recover against them if either the rescuer or the mountain rescue team were negligent. The rescuer and the mountain rescue team had a duty to act towards the hiker according to the appropriate standard of care in order to protect her from unreasonable risk of injury. Because the rescuer and the mountain rescue team were professional "rescuers," they would, at a minimum, be held to a reasonable person standard and, at a maximum, be required to exercise the knowledge and skill of a member of their profession in the same or a comparable community. If the harness latch was not properly fastened to the stretcher, they would have failed to meet the appropriate standard of care and would incur liability to the hiker for negligence. Thus, this is the best answer.

An employee at an automobile engine plant was stopped at the gate of the plant by strikers who had formed a picket line. During the ensuing argument one of the strikers kept hitting the hood of the employee's automobile with a hammer. When the employee refused to turn back, the striker picked up a Molotov cocktail and came running at the employee's automobile from the front while shouting at the employee to get out of the area or the striker was going to let him have it. The employee drove the automobile straight at the striker. The striker leaped out of the way, but the automobile accidentally struck a security guard who had suddenly come up behind the striker in an attempt to prevent him from throwing the Molotov cocktail. In an action by the employee against the striker for trespass to his automobile, the trial judge should

grant judgment as a matter of law for the striker unless the employee presents evidence of some damage to his automobile. Discussion of correct answer:The action of trespass to chattels involves an intentional interference with the plaintiff's possessory interest in personal property. It is the proper action where the defendant's interference does not amount to a substantial interference with the plaintiff's possession of the property (i.e., an exercise of dominion and control over the property) and only consists of inter-meddling with, use of, or damage to the plaintiff's personal property. The measure of damages for a trespass action is the actual amount of loss suffered by reason of the impairment of the property or the loss of its use. The judge should grant judgment as a matter of law for the striker unless the employee can establish a claim for trespass to chattels. One essential element of the plaintiff's case for trespass to chattels is actual harm. Since the striker did not deprive him of possession, the employee must show that the striker damaged the car. The judge can only grant judgment as a matter of law for the employee if he proves all the elements of an action for trespass to chattels. This includes proof of actual harm, which is required to assert an action for trespass to chattels (unlike a case of trespass to real property, where no proof of actual damage is required). Thus, the judge cannot direct a verdict for the plaintiff, even on the issue of liability, until the plaintiff has provided proof of actual damage.

One Sunday morning, a runner was in a 10-kilometer road race. He had made a bet with a co-worker that if he did not finish the race under his best time, he would come to work dressed in children's clothing every day for a week. If he won the bet, however, his co-worker would be the one to suffer such humiliation. Toward the end of the race, the runner realized that he was very close to losing the bet, and he grew very nervous. He decided to cut through a yard along the race route. A signed posted in the yard read, "Beware of dog." The runner glanced quickly around the small yard but saw nothing, and he decided that the sign must either be a fake or that the dog must be inside. The runner jumped the fence and ran across the yard. Just as he was about to hop the other fence to rejoin the race, he was bitten in the foot by a small dog, which clung viciously to his foot. As a result of the bite, the runner lost his pinky toe. The dog, while small, had bitten people before. He filed suit against the owner for having such a vicious dog. The dog owner filed a motion to dismiss. The court should

grant the motion to dismiss, because the runner was an unknown trespasser. Discussion of correct answer:Unknown trespassers may not recover in strict liability for injuries inflicted by domestic animals, even if the animals have "known dangerous propensities." If a trespasser is of another type (known, frequent, or child), the landowner is only responsible if negligent. In this case, the runner was an unknown trespasser, so there is no liability. However, even if he had been known, the landowner would not be liable because the landowner did have a warning sign.

A homeowner owned a townhouse in Midtown. On the lawn in front of his house was a tree stump, which had never been removed after the previous owners removed the top portion of the tree. The roots of the tree stump were growing very close to the sewage lines, and local officials were afraid the roots would cause damage to the pipes. A Midtown ordinance requires landowners to prevent roots from trees on their property from growing too close to the city's sewage pipes. The homeowner engaged a landscaper to remove the stump, and relied on the landscaper's professional judgment to remove the stump in any manner the landscaper deemed appropriate. The landscaper tried to dig out the roots, but realized that the roots were too deep to be dug out. The landscaper had devised a special "burn out" technique for removing especially difficult stumps like this one, which involved setting fire to the stump and burning it down to the roots. The homeowner paid the landscaper in advance for his services, and the next day the landscaper set to work burning out the stump. As he was engaged in burning out the stump, a strong gust of wind carried a billow of black smoke next door to a neighbor's house and blew it through an open window. The landscaper was unable to control the smoke, and it continued to blow into the neighbor's house. The neighbor was not home at the time. The smoke caused significant damage to the furniture in the neighbor's living room. The neighbor asserted claims against the homeowner and the landscaper, and both defendants admit that burning the stump caused the smoke that damaged the neighbor's furniture. In the neighbor's claim against the landscaper, the best defense for the landscaper is that

he could not reasonably foresee that the smoke would billow into the neighbor's living room. Discussion of correct answer:A plaintiff seeking to establish liability in negligence must prove four elements. The plaintiff must prove (1) the defendant owed a legal duty to the plaintiff, (2) the defendant's conduct breached that duty, (3) the defendant's breach of duty was the actual and legal (proximate) cause of injury to plaintiff, and (4) the plaintiff suffered damages as a result of defendant's conduct. The element of "duty" is usually described as an obligation, recognized by law, requiring the defendant to conform to a certain standard of conduct, for the protection of others against unreasonable risk. Where the defendant engages in conduct that is claimed to have injured plaintiff, the issue is whether the defendant had a duty to the plaintiff to conform to a certain standard of conduct. In cases where professionals are the defendants and are liable for an injury that was produced by a tort resulting from their trade, they will be held to a higher standard of conduct. In this case, even though the landscaper could be held to a higher standard, if it was not reasonably foreseeable that the smoke would billow into the neighbor's living room, the landscaper did not breach any duty owed to the neighbor.

A former police officer was the new prosecutor in a small city. Just before he was sworn in, a local drug dealer went public about the former police officer's coming to his "place of business," beating several of his associates with a blackjack, and stealing the drugs for his own use. The former police officer was furious and immediately held a press conference announcing his lawsuit against the drug dealer and making statements about how "all it takes is the word of a drug dealer and people will question a good man's reputation." At trial, the drug dealer will prevail if he can show that

his claims are true. Discussion of correct answer:Truth is a complete defense to any claim of defamation. Therefore, if the drug dealer can prove that his allegations are true, this would provide him with a complete defense in a defamation action.

A homeowner owned a home in a small town. On the lawn in front of his house and within five feet of the public sidewalk there was a large tree. The roots of the tree caused the sidewalk to buckle severely and become dangerous. A local ordinance requires adjacent landowners to keep sidewalks in safe condition. The homeowner engaged a handyman to repair the sidewalk, leaving it to the handyman to decide how the repair should be made. The handyman dug up the sidewalk, cut back the roots of the tree and laid a new sidewalk. Two days after the homeowner had paid the handyman the agreed price of the repair, the tree fell over onto the street and damaged a parked car belonging to a neighbor. The neighbor asserted claims against the homeowner and the handyman, and both defendants admit that cutting the roots caused the tree to fall. The theory on which the neighbor is most likely to prevail against the homeowner is

liability for the handyman's negligence, if, to the homeowner's knowledge, the handyman was engaged in hazardous activity. Discussion of correct answer:A defendant is not liable for torts committed by someone he has engaged as an independent contractor. This is because defendant has no right to control the activity of the contractor. In three situations, contrary to the general rule set forth above, defendant may be held vicariously liable for the torts of an independent contractor. One of these situations is where a contractor engages in inherently dangerous activities (any activity as to which there is a high degree of risk in relation to the particular surroundings, recognizable in advance as requiring special precautions.) Examples include fumigating with poisonous gases, using explosives in an urban setting, etc. If the handyman was engaged in hazardous activity, then the homeowner can be held liable.

One evening, a man asked his cousin if he could borrow the cousin's car to go to a job interview the following day. The location of the interview made driving preferable to taking the bus. The cousin agreed to lend the man his car. The next morning, the man picked up the car from the cousin's house and headed to the interview. However, on the way to the interview, the man decided to stop by a liquor store for a little "fortification." He bought a large bottle of vodka and drank some of its contents in the car as he traveled the remainder of the way to his interview. While the car was parked in the parking lot in front of his prospective employer's place of business, the car was struck by a large van. The driver of the van immediately drove away from the scene and was not located or identified. The collision caused $1,600 damage to the cousin's car. If the cousin sues the man for the damage to his car, the cousin will most likely recover

nothing, unless the damage to the cousin's car was caused by negligence on the part of the man. Discussion of correct answer:At first glance, the facts of the question may appear to raise issues of trespass to chattels or conversion. The intentional tort of trespass to chattels involves an act by the defendant that interferes with the plaintiff's rightful possession of personal property. Conversion involves an act by the defendant that interferes with the plaintiff's rightful possession of personal property to such a degree that it is just to require that the defendant pay the plaintiff the full value of the property. Both torts require the defendant's wrongful interference with the plaintiff's possessory interests. However, under the facts of the question, the man borrowed the car with the owner's permission. The fact that the man used the cousin's car for an unauthorized purpose is irrelevant, because the accident did not occur while the man was engaged in this unauthorized use. Unless the man was negligent in some manner that helped cause the accident, such as in the way that he parked the car in front of his prospective employer's building, he is not liable for the damage to the car. Thus, this is the best answer. If the cousin sues the man for the damage to his car, he will most likely recover nothing, unless the damage to the cousin's car was caused by negligence on the part of the man.

A young baseball fan received a new baseball bat and ball for his eighth birthday as a gift from his friend. The young fan had told his friend earlier that week that he really wanted a traditional wooden bat like the one the friend had. However, the friend bought the young fan a metal bat instead of a wooden one. Angry that he did not receive the bat he wanted, the young fan picked up the baseball and hit it toward his friend, who had turned his back to get a piece of birthday cake. The baseball flew over the friend's head, knocking off his birthday hat. The friend lost his balance, fell to the ground and scraped his chin. In a battery action against the young fan, the friend would most likely

prevail, because the young fan had the requisite intent for battery, and an offensive contact resulted. Discussion of correct answer:A battery is an intentional act that causes a harmful or offensive contact with the plaintiff or with something closely connected to the plaintiff. The defendant must have either intended to cause an immediate harmful or offensive contact or known that such contact was substantially certain to occur. The harmful or offensive contact element is satisfied if the contact would inflict pain or impairment of any body function, or if a reasonable person would regard it as offensive. In this case, the young fan hit the ball at the friend in anger. Although it is unclear whether the young fan intended to injure his friend, such an injury was substantially certain to occur. Therefore, the young fan had the requisite intent for battery. Moreover, even though the young fan did not hit the friend with the baseball, he hit the friend's hat, which was closely connected to the friend's body. This offensive contact caused the friend's fall and his subsequent injury. Therefore, all the elements for battery are present, and the friend would likely prevail.

A thug said to a rival from another neighborhood, "You'd better stay out of my way. The next time I find you around here, I'll beat you up." Two days later, when the rival again visited the thug's neighborhood, the rival saw the thug's identical twin coming toward him. As the two neared each other, the rival saw the twin raise his hand. Mistaking the twin for the thug, and fearing bodily harm, the rival struck the twin. If the twin asserts a claim against the rival and the rival relies on the privilege of self-defense, the rival will

prevail, if a reasonable person under the same circumstances would have believed that the twin would attack him. Discussion of correct answer:A defendant charged with an intentional tort may defend on grounds that she used reasonable force to prevent the plaintiff from engaging in an imminent and unprivileged attack. Even where there is actually no harm threatened against the defendant, she may successfully assert self-defense if a reasonable person in the same circumstances would have believed that she was under attack. Thus, as long as the defendant subjectively (i.e., honestly and in good faith) believes that a sufficient threat exists to justify defensive force, and there is an objective basis for that belief (a reasonable person would believe so under the circumstances), self-defense is available. In this question, it might be reasonable for the rival to have concluded that the twin was the thug (as they were identical twins) and that the twin's act of raising his hand, added to the knowledge that the thug had threatened the rival recently, meant that the rival was about to be attacked. Therefore, a court could conclude that the rival properly acted in self-defense.

The International Ice Dance Company consists of six couples—one couple each from France, Russia, England, Spain, Iran, and China—who perform a show featuring ice skating routines based on folk dances from around the world. Just before a performance in New York one evening, the British dance team had a backstage argument in front of the entire company during which the female skater said to the male skater, "You are sick. You have syphilis." The male skater, in fact, did not have syphilis. He sued his dance partner. In that suit, the male skater will

prevail, if one or more of the other members of the dance company understood English. Discussion of correct answer:For spoken words to be defamatory and constitute slander, there must be publication. Publication of slander occurs only when a third party hears and understands the words (so that the hearer may have a lower opinion of the plaintiff). If they were spoken in a language none of the listeners understood, they were not "published." Thus, this statement is actionable only if one or more of the persons who heard it understands English.

One evening, in a jurisdiction that retains the common-law rules concerning contributory negligence, a visibly intoxicated firefighter entered a bar, where the bartender served him five drinks. The bartender's actions were contrary to the owner's explicit directions to his bartenders, as well as in violation of a state statute making it a misdemeanor to serve alcoholic beverages to any person who is visibly intoxicated. When the bar closed, the firefighter tried to drive home, but he got into a highway accident that caused him internal injuries. He was taken to a city hospital, where he underwent surgery for a ruptured spleen. The nursing staff, who were all extremely overworked, did not change his dressing as frequently as the doctor had ordered, and the firefighter developed an infection that proved to be fatal. In a wrongful death action by the executors of the firefighter's estate against the bartender, the strongest argument on the bartender's behalf would be that

the firefighter was contributorily negligent. Discussion of correct answer:Contributory negligence is a defense that may be raised against the plaintiff even when the defendant's duty of care has been established by statute. Some statutes specifically exclude the intoxicated person as a plaintiff under the Dram Shop Act. Those that do not, presumably, allow the defendant to assert the plaintiff's contributory negligence as a bar to his recovery anyway. This answer is correct.

The manager of a hardware store failed to notice that the post supporting one of the many small lighting fixtures in the hardware store's parking lot had been weakened by age and rust. Several store employees had noticed the problem over the past few months but failed to report it. A handyman, who was preoccupied with his children in the back seat, was not looking where he was going as he was backing his car into a parking space. He hit the weakened post. The post gave way slightly, but the handyman found that he could push the post back so that it looked alright, even though he knew it might give way at any minute. One week later, the post fell and struck a shopper's car as she was entering the lot to shop at the hardware store. The post fell due to the combination of its aged, rusty condition, and the force of the impact of the handyman's car. Either factor alone would not have been sufficient to cause the post to topple. The shopper can recover for the damage to her car in a negligence claim against

the handyman and/or the hardware store for the full amount of her damages. Discussion of correct answer:This is a negligence question. Where two or more defendants, by their concurrent negligent acts, bring about harm to the plaintiff, and it is not possible to separate portions of the harm as being the result of each act, these defendants may be held jointly and severally liable for the entire harm. The damages may be distributed between defendants who are jointly and severally liable for purposes of contribution in a separate action, or by way of impleader among the defendants themselves. However, as to the plaintiff, each defendant is liable to the plaintiff for the full amount. In this case, it is not possible to separate the portions of the harm caused by the handyman and the hardware store, so they will be held jointly and severally liable for the entire harm.

Electric utilities used to use a highly toxic chemical to insulate transformers. As an electric company phased out its old transformers, it drained the chemical into a large storage container in its plant. When the last transformer containing the chemical had been retired, all of the chemical was shipped to a large underground storage facility that the electric company had built in a rural area. The storage facility was built in accordance with specifications, was state of the art at the time that it was built, and was fully expected to contain the chemical indefinitely. When the chemical was stored and the facility sealed, it was not known that, over time, the chemical could corrode the metals of which the storage facility was constructed. The chemical eventually seeped out of the facility into the adjacent ground. Ten years later, a cattle rancher whose ranch was within five miles of the storage facility found that a number of his cattle were dying. He tested his well water and found that it contained a high level of the chemical, which no one besides the electric company had ever used in the area. Unable to find an economically reasonable substitute source of water, he shut down the ranch and sold the land for $100,000 less than its fair market value prior to his discovery of the contamination. If the rancher sues the electric company for the $100,000 of damage he suffered,

the rancher will prevail, because the chemicals which contaminated his well and forced him to terminate ranching came from the storage facility. Discussion of correct answer:The basis of the rancher's suit is nuisance, the unreasonable interference with the use and enjoyment of the plaintiff's land. The activity that causes the unreasonable interference can be the result of intentional conduct, negligence, or abnormally dangerous activity for which a defendant is strictly liable. Therefore, this is the best answer.


Related study sets

Chapter 2: Starting the Planning Process

View Set

Abeka Science 8.5 Types Of Birds

View Set

MODULE 5: MANAGEMENT AND CONTROL OF THE CORP.

View Set

Algebra 1 Lesson 3.1 Graphing Linear Equations - Intercepts

View Set

evolution and the history of life exam 1

View Set

NSG 334 Chapter 29: Growth and Development of the Adolescent

View Set